You are on page 1of 132

1.

"Heart failure" (HF) cells are seen in:


a. Kidneys
b. Lungs
c. Pancreas
d. Pericardium

2. Metaplasia
a. Is an increase in the number and size of cells in a tissue.
b. Is the process that occurs in Barretts oesophagitis.
c. Is typically an irreversible process.
d. Preserves mucus secretion in the respiratory tract.

3. Dystrophic calcification are calcifications seen in:


a. Normal tissues
b. Dead tissues
c. Liver and Kidney
d. None of the above

4. Deposits of amyloid in whole organs can be diagnosed by histological staining nolan x soju
with a. Congo red
b. PAS
c. Methylviolet
d. Iodine and dilute sulfuric acid
e. Toluidine blue

5. A 3-year-old child has been diagnosed with metabolic disorder and has developed
hepatic failure. The left lobe of an adult donor liver is used as a transplant. A year
later, the size of each liver in donor and recipient is greater than at the time of
transplantation. Which of the following cellular alterations is most likely to explain
this phenomenon? a. Metaplasia
b. Hypoplasia
c. Dysplasia
d. Hyperplasia
6. During acute inflammation: histamine-induced increased vascular permeability
causes the formation of exudates (inflammatory edema). Which one of the listed
cell types is the most likely source of the histamine that causes the increased
vascular permeability?
a. Endothelial cells
b. Fibroblasts
c. Lymphocytes
d. Mast cells
7. Which of the following mediators are involved in the phase of vascular changes
during acute inflammation? a. IL-12
b. Histamine
c. Substance P
d. Thrombin

8. Which of the following best reflects the order of nuclear loss in cell death?
a. Karyopyknosis, Karyiorrhexis, Karyiolysis
b. Kartolysis, Kariorrhexis, Karyopyknosis
c. Kariopyknosis: Kariolysis, Kariorrhexis
d. Kariolysis, Kariopyknosis, Kariorrhexis

9. Reversible cell injury is characterised by all of the following, EXCEPT?


a. Detachment of ribosomes nolan x soju
b. Nuclear chromatin clumping
c. Cellular swelling
d. lysosomal rupture

10. Which of the following is NOT a malignant tumor?


a. Osteosarcoma
b. Melanoma
c. Leukemia
d. Lipoma

11. Which of the following cells form multinucleated giant cells?


a. Plasma cells
b. Macrophages
c. Neutrophils
d. None of the above
12. Which of the following substances accumulate in cells during aging and chronic
diseases?
a. Glycogen
b. Lipofuscin
c. Melanin

13. A 4-year-old boy falls on a rusty nail and punctures his skin. The wound is cleaned
and covered with sterile gauze. Which of the following is the initial event in the
healing process?
a. Accumulation of acute inflammatory cells
b. Deposition of proteoglycans and collagen
c. Formation of a fibrin clot
d. Macrophage-mediated phagocytosis of cellular debris

14. Irreversible cell injury is characterized by


a. Dispersion of ribosomes.
b. cell swelling.
c. lysosomal rupture
d. cell membrane defects.
e. nuclear chromatin clumpmg

15. Which of the following cell types belong to stable cell population? nolan x soju
a. Neurons
b. Cardiac cells
c. Surface epithelial cells
d. Hepatocytes

16. Which of the following mediators most likely contributes to giant cell formation?
a. Interferon y
b. Histamine
c. IL-I
d. None of the above

17. Which of following acts as an opsonin in phagocytosis?


a. C3b
b. LTB4
c. IL-8
d. None of the above

18. An S7-year-oId male develops worsening heart failure. Workup reveals decreased
left ventricular filling due to decreased compliance of the left ventricle. Two
months later the patient dies: and postmortem sections reveal deposits of
eosinophilic, Congo red—positive material in the interstitium of his heart. When
viewed under polarized light, this material displays an apple-green birefringence.
1%at is the correct diagnosis? a. Amyloidosis
b. Glycogenosis
c. Hemochromatosis
d. Sarcoidosis

19. Which of the following statements regarding teratomas is INCORRECT:


a. They can arise in the ovary
b. They arise from totipotential germ cells
c. A sarcomatous element is usually present
d. Tissues resembling those from an embryo can be seen

20. Which of the following describes the pathogenesis of edema?


a. Decreased protein synthesis
b. Increased accumulation of unmetabolized carbon in macrophages
c. High protein intake nolan x soju
d. None of the above

21. Which of the following is an example of physiologic apoptosis?


a. Involution of hormone-dependent tissues upon hormonal deprivation
b. Elimination of self-reactive lymphocytes
c. Cell loss in proliferating cell population
d. All of the above

22. Dysplasia
a. Is a feature of mesenchymal cells.
b. Inevitably progresses to cancer.
c. Is characterised by cellular pleomorphism.
d. Is not associated with tissue architectural abnormalities.
e. Is the same as carcinoma in situ.

23. A 25-year-oldmachinistisinjuredbyametalsliverinhis left hand- Over the next few


days, the wounded area becomes reddened, tender, swollen, and feels warm to
the touch. Redness at the site of injury in this patient is caused primarily by which
of the following mechanisms? a. Hemorrhage
b. Neutrophil margination
c. Vasoconstriction
d. Vasodilation

24. Which of the following best reflects the order of events in acute inflammation?
a. Adhesion: Chemotaxis, Transmigration: Phagocytosis
b. Margination, Chemotaxis, Phagocytosis: Adhesion
c. Rolling, Chemotaxis, Transmigration, Phagocytosis
d. Rolling, Adhesion, Chemotaxis, phagocytosis

25. "Nutmeg" Liver occurs in:


a. Chronic venous congestion
b. Hepatocellular carcinoma
c. Jaundice
d. None of the above

26. Which of the following is a good example of type Ill hypersensitivity reaction? nolan x soju
a. Tuberculosis
b. Serum sickness
c. Asthma
d. None of the above

27. . Which of the following is true for metaplasia?


a. Metaplasia is an increase in the size of a tissue or organ caused by an increased
number of constituent cells
b. Metaplasia is reversible and the tissue reverts to its normal state after the
stimulus or irritant has been removed
c. Metaplasia results from an action of mechanical stimuli (increased workload)
28. Which one of the listed changes correctly describes the pathophysiology involved
in the production of pulmonary edema in patients with congestive heart failure? a.
Decreased plasma oncotic pressure
b. Endothelial damage
c. Increased hydrostatic pressure
d. Lymphatic obstruction

29. Which of the following cell types differentiates into morphologically distinct cells
capable of immunoglobulin production? a. Neutrophils
b. Basophils
c. B cells
d. T cells
e. Plasma cells

30. Which of the following is a characteristic feature of malignant tumors?


a. They are slow growing
b. They never metastasize
c. They are invasive
d. None of the above

31. Which of the following is NOT a malignant tumor?


a. Melanoma
b. Chondrosarcoma
c. Papilloma
d. Lymphoma
nolan x soju

32. A histopathologist reports fat necrosis after examning a slide. Fat necrosis might
be found in which one of the following situations? a. Brain injury
b. Muscle injury
c. Trauma to the breast
d. Trauma to the bowel
33. A woman who is allergic to cats visits a neighbour who has several cats. During the
visit, she inhales cat dander, and within minutes: she develops nasal congestion
with abundant nasal secretions. 1%ich of the following substances is most likely to
produce these findings? a. Histamine
b. Substance P
c. IL-12
d. All of the above

34. Wound healing is affected by:


a. Infections
b. Poor nutrition
c. Age
d. All of the above

35. Regarding oedema


a. Hypoproteinaemia is the most common cause of systemic oedema.
b. Hepatic cirrhosis is the most common cause of hypoproteinaemia.
c. Facial oedema is a prominent feature of anasarca.
d. Hereditary angioneurotic oedema involves skin only.
e. Infection does not cause pulmonary oedema.

36. Fluid that collects during acute inflammation and that has a protein content
exceeding 3 g/dl and a specific gravity exceeding 1.015 is referred a. Edema
b. An effusion
c. Answer is C or D

nolan x soju
37. Which of the following terms refer to a malignant tumor of the cartilage?
a. Chondroma
b. Fibrosarcoma
c. Chondrosarcoma
d. Osteosarcoma
e. Osteoma
f. Adenocarcinoma
38. Activation of which of the following cytoplasmic enzymes is most likely seen in
apoptosis?
a. Caspases
b. Ribonucleases
c. Telomerases
d. None of the above
39. Which feature in a tumor suggests that it is likely to be a benign neoplasm?
a. Circumscription
b. Lack of nuclear pleomorphism
c. Low mitotic count
d. All of the above

40. Klinefelter's syndrome does not include which of the following conditions?
a. Mental retardation
b. Hypogonadism
c. Infertility
d. Gynecomastia
e. Absence of germ cells

41. Human immunodeficiency virus (HIV) has a special affinity for infecting which type
of cell?
a. A-CD8+ T cell
b. CD4+ T cell
c. Natural killer cell
d. Dendritic cell

42. In reversible cell injury all are true except


a. ATP depletion is responsible for acute cellular swelling
b. Can cause myocardial cells to cease contraction within 60 seconds
c. ATP is generated anaerobically from creatine phosphate
d. Mitochondrial swelling and degranulation of ER are hallmarks of irreversible
cellular
e. Is associated with myelin figures nolan x soju

43. Metastatic calcification:


a. Is encountered in areas of necrosis
b. Can occur with normal serum levels of calcium
c. Can be associated with metaplasia
d. Is commonly associated with hyperparathyroidism
44. White infarcts occur in the
a. A. small intestine.
b. Kidney
c. Lung.
d. sigmoid colon-
e. Oesophagus.

45. What are the outcomes of thrombi?


a. Resolution: Propagation, Organisation
b. Hyperplasia: Propagation: Embolisation
c. Embolization, Dysplasia, Organisation
d. Recanalization, Infiltration, Accumulation

46. Fatty change:


a. Is irreversible
b. Does not impair cellular function
c. Is most commonly due to diabetes
d. Is caused by alcohol

47. A 35-year-old woman had a firm nodule palpable on the dome of the uterus six
years ago on routine examination. Then discovered. She is asymptomatic. The
uterine nodule most likely represents a(an): a. Adenocarcinoma
b. Rhabdomyoma
c. Hamartoma
d. Leiomyoma

48. In tumor invasion & dissemination:


a. Extracellular matrix cleavage products are growth promoting & chemotactic for
tumor cells.
b. Tumor invasion through the extracellular matrix is by progressive expansile nolan x soju
growth
c. Tumor cells downregulates their adhesion molecules to reduce adhesion to
each other during vascular dissemination
d. Organ selective homing of tumor cells are purely due to the anatomical location
of the primary tumor.
e. All of the above

49. A 36-year-old female presents with increased trouble swallowing. Physical


examination finds hypertension and sclerodactyly. A skin biopsy reveals dermal
fibrosis with an absence of adnexal structures, while laboratory examination finds
an autoantibody against DNA topoisomerase (anti-ScI-70). What is the best
diagnosis for this individual?
a. Mixed connective tissue disorder
b. Systemic sclerosis
c. Sjögren's syndrome

a.
d. Systemic lupus erythematosus

50. In apoptosis
It involves physiologic and pathologic stimuli
b. Selectins
c. Integrins
d. Immunoglobulins
56. In primary wound healing:
a. Macrophage infiltration occurs at 12hrs
b. Wound strength is 50% of normal at the end of first 2 days
c. Type I collagen is replaced by type Ill collagen
d. None of the above

57. What are the main opsonins involved in phagocytosis?


a. Nitric oxide and Histamine
b. IL-8 and Leukotriene B4
c. IgG and C3b
d. Serotonin and C3b

58. Ten minutes after being stung by a wasp: a 30-year-old male develops multiple
patches of red, irregular skin lesions over his entire body. These lesions (urticaria)
are pruritic: and new crops of lesions occur every day. This response is primarily
the result of liberation of specific vasoactive substances by the action of: a.
Activated T lymphocytes on smooth muscle cells
b. IgA on basophils and mast cells nolan x soju
c. IgA on lymphocytes and eosinophils
d. lgE on basophils and mast cells

59. Each classic term describing acute inflammation is correctly matched with its cause
EXCEPT
a. Rubor — vasodilatation
b. Dolor — inflammatory cells infiltrating blood vessels
c. Color — increased blood flow
d. Tumor — exudation of fluid and cells into extravascular tissues

a.
60. Regarding fatty change, which statement is TRUE:
a. It may result from protein malnutrition
b. It may result from anoxia
c. It may result from systemic sclerosis
d. Fatty deposits are seen in all hepatocytes in alcoholic fatty liver

61. The endogenous pigment (s) NOT derived from hemoglobin is (are) Hemosiderin
b. Lipofuscin

62. Which of the following pigments will accumulate as the result of peroxidation of
cell membranes?
a. Bilirubin
b. Lipochrome
c. Melanin
d. Hemomelanin
e. hemosiderin

63. Which of the following is a good example of type II hypersensitivity reaction?


a. Transfusion reaction
b. Autoimmune hemolyic anemia
c. Goodpasture syndrome
d. All of the above

64. Which of the following is NOT a sign of anaplasia?


a. cytoplasmic & nuclear pleomorphism
b. large nucleoli & hyperchromic chromatin
c. presence of mitotic figures nolan x soju
d. disturbed orientation of cells
e. increased nuclear & cytoplasmic ratio

65. Transudation may occur due to the following EXCEPT

a.
a. Increased intravascular hydrostatic pressure
b. Decreased intravascular oncotic pressure
c. Lymphatic Obstruction
d. Inflammation

66. Malignant tumors has the following characteristics:


a. All malignant tumors are poorly differentiated
b. All malignant tumors metastasize
c. Anaplasia is a consequence of de-differentiation
d. Malignant tumors may develop capsules
e. Malignant tumors are always more rapidly proliferative than benign tumors

67. Metaplasia is seen in all of the following EXCEPT


Respiratory epithelium of smokers

a.
b. Vitamin A excess
c. Barrett's oesophagitis

68. The inability to distinguish between self-cells and non-self-cells may lead to:
a. Autoimmune diseases
b. Cancer
c. Immunodeficiency syndromes
d. Dysplasia

69. Which of the following is a feature of necrosis?


a. Caspase activation
b. Karyorrhexis
c. Chromatin condensation
d. None of the above

70. What is the most common site of origin of thrombotic pulmonary emboli?
a. Deep leg veins
b. Lumen of left ventricle
c. Lumen of right ventricle
d. Mesenteric veins nolan x soju

71. Severe cell injury due to ischemia is characterized by


a. increased cellular capacity for oxidative phosphorylation
b. a loss of intracellular calcium
c. shrinkage of mitochondria
d. decreased intracellular pH
e. decreased intracellular lactate

72. . Metastasis
a. Unequivocally proves malignancy.
b. Is proven by lymph node enlargement adjacent to a tumor.
c. Of breast cancer is usually to supraclavicular nodes.
d. Is the most common presentation of melanoma
e. all of the above
73. Severe generalized edema is called:
a. Neoplasia
b. Anasarca
c. Pitting edema
d. Myxoedema

74. Which of the following examples represent metaplasia?


a. Increase in size of the uterus during pregnancy
b. Thymic involution in elderly
c. Prostatic enlargement in men over 50
d. Changing the esophageal epithelium into gastric epithelium due to GERD4

75. The most characteristic feature of granulation tissue is:


a. Growth of fibroblasts and capillaries
b. Formation of granuloma
c. Neoplastic changes of the epithelium
d. None of the above

76. . Which of the following terms refer to a malignant tumor of a smooth muscle?
a. Leiomyoma
b. Leiomyosarcoma
c. Adenoma
d. Rhabdomyosarcoma nolan x soju

77. All of the following associations are correctly paired EXCEPT


a. Fragile X — mental retardation
b. XXY — male infertility
c. XXXXY. — Patau's syndrome
d. 5p — characteristic cry

78. Which of the following terms refer to a benign tumor of a skeletal muscle?
a. Rhabdomyoma
b. Rhabdomyosarcoma
c. Fibroma
d. Fibrosarcoma
79. The first vascular response to Injury is
a. slowing of the circulation
b. Arteriolar vasoconstriction.
c. capillary engorgement
d. recruitment of vascular beds-
e. venular dilation.

80. Which of the following predisposes to thombogenesis?


a. Stasis of blood
b. Endothelial injury
c. Turbulence of blood
d. All of the above

81. In the inflammatory response to this infection, which of the following cells removes
the bacteria by phagocytosis? a. Macrophages
b. Mast cells
c. Endothelial cells
d. T regulatory cells

82. All clinical features listed below are usual for Turner's Syndrome: EXCEPT
a. Primary amenorrhea
b. Infertility
c. Most over 6 feet tall
d. Webbing of neck
e. Cubitus valgus
nolan x soju

83. Which of the following is true for hyperplasia?


a. Bone marrow hyperplasia in high altitudes is an example of physiologic
hyperplasia
b. Hyperplasia is always pathological
c. Hyperplasia is a decrease in the size of a tissue or an organ caused by a
decreased number of constituent cells
d. Hyperplasia is an example of an Irreversible cell injury

84. Which of the following can be a complication of thrombosis?


a. Neoplastic transformation of the vessel wall
b. Embolization
c. Neovascularisation
d. None of the above

85. Which of the following statements regarding tumorigenesis is TRUE?


a. All malignant tumors are monoclonal.
b. Mutant alleles of proto-oncogenes can incur cell transformation even in
presence of their normal counterparts (ie. It is a dominant phenomenon)
c. Both mutant alleles of anti-oncogenes must exist for cell transformation to
occur
(ie. It is a recessive phenomenon)
d. Phenotypic attributes of a tumor is acquired in a stepwise fashion
e. All of the above

86. Which of the following mediators of inflammation cause pain?


a. IL-8 and C3b
b. Serotonin and Leukotriene B4
c. Prostaglandins and Bradykinin
d. Bradykinin and Histamine

87. Which of the following neoplasms is associated with HIV infection?


a. Kaposi sarcoma
b. Liposarcoma
c. Hemangioma
d. Melanoma

88. Which of the following is a characteristic feature of benign tumors? nolan x soju
a. Mostly they are undifferentiated
b. They can invade the tissues around
c. They usually resemble the tissue of origin
d. All of the above

89. What is the outcome of necrosis?


a. Calcification
b. Repair
c. Complete restitution regeneration
d. All of the above

90. Which of the following is a hallmark of cancer?


a. Evasion of apoptosis
b. Insensitivity to growth inhibitory signals
c. Ability to metastasize
d. All of the above

91. Which is NOT evidence of irreversible cell injury?


a. Acute cell swelling ("cellular edema")
b. Nuclear pyknosis
c. Rupture of the lysosomes
d. All the above are evidence of irreversible injury

92. Squamous metpasia may result from:


a. Smoking
b. Ageing
c. Hormone deprivation

93. The term that describes the adherence of neutrophils and monocytes to the
vascular endothelium prior to movement into the extravascular space is a.
Margination
b. Diapedesis
c. Pavementing
d. Emigration
e. Clotting

94. Which of the following viral infections is associated with cervical cancer
development? a. EBV nolan x soju
b. CMV
c. HPV
d. 1--1N

95. Neoplastic growth has the following characteristics:


a. All cells in a tumor is in actively replicating phase of the cell cycle as there is a
loss of proliferative control
b. Tumors with small growth fraction is refractive to chemotherapy
c. Tumor growth is associated with shortened cell cycle
d. D. Frequency of mitoses in a neoplasm is not reflective of its growth rate
e. All of the above are correct.

96. Metastasis refers to:


a. Carcinoma in situ
b. The spread of cancer cells around the body
c. Well-differentiated cells
d. An aspect of benign tumors

97. Which of the following terms refer to a malignant tumor of a mesenchymal origin?
a. Sarcoma
b. Teratoma
c. Carcinoma
d. Adenoma

98. Dystrophic calcification are calcifications seen in:


a. Normal tissues
b. Dead tissues

99. Hyperplasia
a. Occurs after partial hepatectomy
b. Refers to an increase in the size of the cells
c. Is always pathological
d. Often occurs in cardiac and skeletal muscle
e. Usually progresses to cancerous proliferation

100. In relation to hypertrophy, which of the following statements is


CORRECT? a. It is triggered by specific hormonal stimulation nolan x soju
b. It increases the function of an organ
c. It is usually pathological
d. It occurs after panial hepatectomy
101. All of the following vascular changes are observed in acute inflammation, except:
a. Vasodilation
b. Stasis of blood
c. Increased vascular permeability
d. Decreased hydrostatic pressure
1. Human immunodeficiency virus (HIV) has a special affinity for infecting which type of cell?
- CD8+ T cell
- CD4+ T cell
- Natural killer cell
- Dendritic cell
- B cell

2. A 25-year-old woman breastfed her first baby for almost 1 year with no difficulties and no
complications. Which of the following cellular that occurred in the breast during pregnancy
allowed her to nurse the infant for this period of time?
- Ductal epithelial metaplasia
- Epithelial dysplasia
- Lobular hyperplasia
- Stromal hypertrophy

3. Which of the following substances accumulate in cells during aging and chronic disease?
- Glycogen
- Lipofuscin
- Melanin
- None of the above

4. Neoplastic growth has the following characteristics nolan x soju


- All cells in a tumor is in actively replicating phase of the cell cycle as there is a loss of
proliferative control
- Tumors with small growth fraction is refractive to chemotherapy
- Tumor growth is associated with shortened cell cycle
- Frequency of mitoses in a neoplasm is not reflective of its growth rate
- All of the above are correct

5. In reversible cell injury all are true except


- ATP depletion is responsible for acute cellular swelling
- Can cause myocardial cells to cease contraction within 60 seconds
- ATP is generated anaerobically from creatine phosphate
- Mitochondrial swelling and degranulation of ER are hallmarks of irreversible cellular injury
- Is associated with myelin figures

6. Which of the following cell types belong to permanent cell population?


- Surface epithelial cell
- Hepatocytes
- Cuboidal cells in ducts of exocrine glands
- None of the above

7. Metastatic calcification
- Is encountered in areas of necrosis
- Can occur with normal serum level of calcium
- Can be associated with metaplasia
- Is commonly associated with hyperparathyroidism

8. Which of the following is NOT a malignant tumor?


- Melanoma
- Chondrosarcoma
- Papilloma
- Lymphoma

9. The endogenous pigment (s) NOT derived from hemoglobin is (are)


- Hemosiderin
- Lipofuscin
- Hematin
- Hematoidin

10. Which of the following is true for metaplasia?


- Metaplasia is an increase in the size of a tissue or organ caused by an increased number of
constituent cells
- Metaplasia is reversible and the tissue reverts to its normal state after the stimulus or nolan x soju
irritant has been removed
- Metaplasia results from an action of mechanical stimuli (increased workload)
- A good example of metaplasia is thymic involution

11. What is the most common site of origin of thrombotic pulmonary emboli?
- Deep leg veins
- Lumen of left ventricle
- Lumen of right ventricle
- Mesenteric veins

12. In relation to atrophy, which of the following statements is FALSE?


- Decreased autophagic vacuoles
- Decreased myofilaments
- Decreased smooth endoplasmic reticulum
- None of the above

13. Anaplasia is not characterized by


- Pleomorphism
- Abundant nuclear DNA
- A nuclear cytoplasm of 1 : 6
- Coarsely clumped chromatin
- Lack of differentiation

14. A 64-year-old male present with symptoms of anemia. On workup you discover that the
patient has been losing blood from the GI tract secondary to a tumor mass in his colon. The
pathology report from a biopsy specimen indicates that this mass is an invasive
adenocarcinoma. Which one of the listed descriptions best describes he most likely
histologic appearance of this tumor?
- A uniform proliferation of fibrous tissue
- A disorganized mass of proliferating fibroblast and blood vessels
- A disorganized mass of smooth muscle
- A disorganized mass of cells forming glandular structures

15. Which of the following examples represent metaplasia?


- Increase in size of the uterus during pregnancy
- Thymic involution in elderly
- Prostatic enlargement in men over 50
- Changing the esophageal epithelium into gastric epithelium due to GERD

16. Which of the following is NOT a feature of reversible cell injury?


- Cellular swelling nolan x soju
- Reduction of ATP synthesis
- Clumping of nuclear chromatin
- Defects in cell membrane

17. Which one of the listed statements best describes the process called chemotaxis?
- Abnormal fusion of phagosomes to primary lysosomes
- Attachment of chemicals to extracellular material to increase phagocytosis
- Dilation of blood vessels
- Movement of cells toward a certain site or source

18. Which of the following terms refer to a malignant tumor of a mesenchymal origin?
- Sarcoma
- Teratoma
- Carcinoma
- Adenoma

19. The first vascular response to injury is


- Slowing of the circulation
- Arteriolar vasoconstriction
- Capillary engorgement
- Recruitment of vascular beds
- Venular dilation

20. Platelet-derived growth factor is best-known for


- Activating fibroblast in wound healing
- Activating the complement cascade
- Aiding the maturation of granulomas
- Causing epithelial metaplasia

nolan x soju

1. Which of the following is NOT a malignant tumor?


- Osteosarcoma
- Melanoma
- Leukemia
- Lipoma

2. What type of antibody is most abundant in the serum and function as an opsonin?
- IgG
- IgM
- IgD
- IgE

3. Which of the following pigments will accumulate as the result of peroxidation of cell
membranes?
- Bilirubin
- Lipochrome
- Melanin
- Hemomelanin
- Hemosiderin

4. Ten minutes after being stung by a wasp, a 30-year-old male develops multiple patches of
red, irregular skin lesion over his entire body. These lesions are pruritic and new crops nolan x soju
of lesions occur every day. This response is primarily the result of liberation of specific
vasoactive substances by the action of
- Activated T lymphocytes on smooth muscle cells
- IgA on basophils and mast cells
- IgA on lymphocytes and eosinophils
- IgE on basophils and mast cells

5. Metastasis refers to
- Carcinoma in situ
- The spread of cancer cells around the body
- Well-differentiated cells
- An aspect of benign tumors

6. Activation of which of the following cytoplasmic enzymes is most likely seen in apoptosis?
- Caspases
- Ribonucleases
- Telomerases
- None of the above
7. Severe cell injury due to ischemia is characterized by
- Increased cellular capacity for oxidative phosphorylation
- A loss of intracellular calcium
- Shrinkage of mitochondria - Decreased intracellular pH
- Decreased intracellular lactate

8. Which of the following is an example of physiologic apoptosis?


- Involution of hormone-dependent tissues upon hormonal deprivation
- Elimination of self-reactive lymphocytes
- Cell loss in proliferating cell population
- All of the above

9. Klinefelter’s syndrome does not include which of the following condition?


- Mental retardation
- Hypogonadism
- Infertility
- Gynecomastia
- Absence of germ cells

10. Which of the following terms to a malignant tumor of testicular epithelium?


- Seminoma nolan x soju
- Mesothelioma
- Hemangioma
- Lymphoma

11. Dysplasia
- Is a feature of mesenchymal cells
- Inevitably progresses to cancer
- Is characterized by cellular pleomorphism
- Is not associated with tissue architectural abnormalities
- Is the same as carcinoma in situ

12. What are the main opsonins involved in phagocytosis?


- Nitric oxide and histamine
- IL-8 and Leukotriene B4
- IgG and C3b
- Serotonin and C3b

-
-
-

13. In tumor invasion & dissemination


Extracellular matrix cleavage products are growth promoting & chemotactic for tumor cells
Tumor invasion through the extracellular matrix a by progressive expansile growth Tumor
cells downregulates their adhesion molecules reduce adhesion to each other during
vascular dissemination
- Organ selective boming of tumor cells are purely due to the anatomical location of the
primary tumor
- All of the above

14. Severe generalized edema is called


- Neoplasia
- Anasarca
- Pitting edema
- Myxoedema

15. The inability to distinguish between self-cells and non-self-cells may lead to
- Autoimmune diseases
- Cancer
- Immunodeficiency syndromes
- Dysplasia

16. Concerning the repair of a well opposed clean surgical incision


- There is an initial inflammatory response
- 15% of original tissue strength is attained at one week nolan x soju
- Granulation tissue does not occur
- New collagen begins to accumulate after the first week
- Dermal appendages destroyed by the incision usually recover

17. Which feature is a tumor suggests that it is likely to be a benign neoplasm?


- Circumscription
- Lack of nuclear pleomorphism
- Low mitotic count
- All of the above

18. Malignant tumor has the following characteristics


- All malignant tumors are poorly differentiated

-
-
-

- All malignant tumor metastasize


- Anaplasia is a consequence of de-differentiation
- Malignant tumors may develop capsules
- Malignant tumors are always more rapidly proliferative than benign tumors

19. The most characteristic feature of granulation tissue is


Growth of fibroblast and capillaries
Formation of granuloma
Neoplastic changes of the epithelium
- None of the above

20. Which of the following auto-antibodies are diagnostic for Systemic Lupus Erythematosus
(SLE)?
- Antibodies to double-stranded DNA
- Antibodies to tissue transglutaminase
- Antibodies to ribonucleoproteins
- Antibodies to TSH receptor

21. In an experiment bacteria are introduced into a preferred tissue preparation. Leukocytes
leave the vasculature and migrate to the site of bacterial inoculation. The movement of
these leukocytes is most likely to be mediated by which of the following substance?
- IL-12
- Prostaglandins
- Chemokines
- Histamine

22. Which of the following mediators are involved in the phase of vascular changes during acute
inflammation?
- IL-12 nolan x soju
- Histamine
- Substance P
- Thombin

23. Dystrophic calcification are calcification seen in


- Normal tissues
- Dead tissues
- Liver and kidney
- None of the above
-
-
-

24. Which of the following is a characteristic feature of malignant tumors?


- They are slow growing
- They never metastasize
- They are invasive
- None of the above

25. In the inflammatory response to this infection, which of the following cells removes the
bacteria by phagocytosis?
Macrophages
Mast cells
Endothelial cells
- T regulatory cells

26. The cellular accumulation in case of fatty change disorder of liver, may result from all of the
following factors EXCEPT
- Increased transport of precursor substance to cell
- Overproduction of accumulated material by cell
- Decreased mobilization of accumulated material by cells
- Decreased utilization of accumulated material by cells
- Irreversible damage to mitochondria

27. A 25-year-old machinist is injured by a metal silver in his left hand. Over the next few days
the wounded area becomes reddened, tender, swollen, and feels warm to the touch.
Redness at the site of injury in this patient is caused by which of the following mechanism?
- Hemorrhage
- Neutrophil margination
- Vasoconstriction
- Vasodilation

28. Which of the following cells from multinucleated giant cells?


- Plasma cells nolan x soju
- Macrophages
- Neutrophils
- None of the above

29. Which of the following cell types belong to liable cell population?
-
-
-

- Cardiac cells
- Surface epithelial cells
- Neurons
- None of the above

30. All of the following are morphological feature of apoptosis EXCEPT


- Cell swelling
- Chromatin condensation
- Lack of inflammation
- Phagocytosis of apoptotic bodies
- Formation of cytoplasmic blebs

31. By definition granulomas are composed of Collagen

-
-

Caspase activation
- Karyorrhexis
- Chromatin condensation - None of the above

39. Which of the following enzymes degrade basement membranes of vessels during
metastasis?
- Granzymes
- Telomerases
- Collagenases
- All of the above

40. Which of the following is a characteristic feature of benign tumors?


- Mostly they are undifferentiated
- They can invade the tissues around
- They usually resemble the tissue of origin
- All of the above

41. “Nutmeg” Liver occurs in


- Chronic venous congestion
- Hepatocellular carcinoma
- Jaundice nolan x soju
- None of the above

42. Regarding chemical mediators of inflammation


- Histamine is derived from plasma
- Serotonin is preformed in mat cells
- Nitric oxide is preformed in leukocytes
- The k….. system is activated in platelets
- C3b is within macrophages
-

Antibodies to ribonucleoproteins
- Antibodies to TSH receptor

7. Following coronary artery occlusion, the earliest morphologic finding that closely correspond
to severe loss of membrane integrity and irreversible cell injury is.
- Swelling and vacuolization of mitochondria and the appearance of large mitochondrial
densities
- Cell swelling
- Depletion of cellular glycogen stores (as demonstrated by PAS stain)
- Blebs on cell membranes
- Clumping and condensation of nuclear chromatin

8. Which of the following is true for metaplasia?


- Metaplasia is an increase in the size of a tissue or organ caused by an increased number of
constituent cells
- Metaplasia is reversible and the tissue reverts to its normal state after the stimulus or
irritant has been removed
- Metaplasia results from an action of mechanical stimuli (increased workload)
- A good example of metaplasia is thymic involution

9. Which of the following terms refers to a malignant tumor of a mesenchymal origin?


- Sarcoma nolan x soju
- Teratoma
- Carcinoma
- Adenoma

10. Which of the following terms refers to a malignant tumor of the cartilage?
- Chondroma
- Fibrosarcoma
- Chondrosarcoma
- Osteosarcoma
- Osteomia
- Adenocarcinoma

11. Which of the following neoplasms is associated with HIV infection?


- Kaposi sarcoma
- Liposarcoma
-

- Hemangioma
- Melanoma

12. Fatty change


Is irreversible
- Does not impair cellular function
- Is most commonly due to diabetes - Is caused by alcohol

13. Thrombosis due to hypercoagulability is seen in


- Pregnancy
- Woman taking oral contraceptives
- Cancer
- All of the above

14. The term that describes the adherence of neutrophils and monocytes to the vascular
endothelium prior to movement into the extravascular space is
- Margination
- Diapedesis
- Pavementing
- Clotting

15. White infarcts occur in the


- Small intestine
- Kidney
- Lung nolan x soju
- Sigmoid colon
- Oesophagus

16. All of the following abnormalities may be seen in Marfan’s syndrome EXCEPT
- Ectopia lentis
- Dissecting aneurysm of the aorta
- Mitral valve prolapse
- Sclerodactyly
-
- Aortic valvular insufficiency
13. Which of the following cell types are most closely associated with chronic inflammatory
processes?
- Neutrophils
- Plasma cells
- Eosinophils
- All of the above

14. Fluid that collects during acute inflammation and that has a protein content exceeding 3
g/dl and a specific gravity exceeding 1.015 id referred
- Edema
- An effusion
- A transudate
- Serum
- An exudate

15. All of the following abnormalities may be seen in Marfan’s syndrome EXCEPT
- Ectopia lentis
- Dissecting aneurysm of the aorta
- Mitral valve prolapse
- Sclerodactyly
- Aortic valvular insufficiency

nolan x soju
16. Hyperplasia
- Occurs after partial hepatectomy
- Refers to an increase in the size of the cells
- Is always pathological
- Often occurs in cardiac and skeletal muscle
- Usually progresses to cancerous proliferation

17. Which one of the listed changes correctly describes the pathophysiology involved in the
production of pulmonary edema in patient with congestive heart failure?
- Decreased plasma oncotic pressure
- Endothelial damage
- Increased hydrostatic pressure
- Lymphatic obstruction

18. Which of the following is an example hypertrophy?


- Increased respiratory epithelium in response to vitamin A deficiency
- Increased size of female breast during puberty
- Increased size of female uterus during pregnancy
- Increase in size after partial hepatectomy

19. Squamous metaplasia may result from


- Smoking
- Ageing
- Hormonal deprivation

20. Irreversible cell injury is characterized by


- Dispersion of ribosomes
- Cell swelling
- Lysosomal rupture
- Cell membrane defects
- Nuclear chromatin clumping

21. Each classic term describing acute inflammation is correctly matched with its cause EXCEPT
- Rubor – vasodilation
- Dolor – inflammatory cells infiltrating blood vessels
- Color – increased blood flow
- Tumor – exudation of fluid and cells into extravascular tissue

22. Which of the following terms to a malignant tumor of a smooth muscle?


- Lesomyoma
- Lesomyosarcoma
- Adenoma nolan x soju
- Rhabdomyosarcoma

23. A woman who is allergic to cats visits a neighbour who has several cats. During the visit,
she inhales cat dander, and within minutes, she develops nasal congestion with abundant
nasal secretions. Which of the following substances is most likely to produce these findings?
- Histamine
- Substance P
- IL-12
- All of the above

24. Which of the following chemical mediators act in chemotaxis during acute inflammation? -
Nitric oxide
- IL – 12
- Serotonin
- None of the above

25. Which of the following mediators most likely contributes to giant cell formation?
- Interferon ỿ
- Histamine

38. A 36-year-old female presents with increased trouble swallowing. Physical examination
finds hypertension and sclerodactyly. A skin biopsy reveals dermal fibrosis with an absence
of adnexal structures, while laboratory examination finds an autoantibody against DNA
topossomerase (anti-cl-70). What is the best diagnosis for this individual?
- Systemic sclerosis
- Mixed connective tissue disorder
- Sjogren’s syndrome
- Systemic lupus erythematosus

39. Which of the following terms refer to a benign tumor of a skeletal muscle?
- Rhabdomyoma
- Rhabdomyosarcoma
- Fibroma
- Fibrosarcoma

40. Which of the following cells form multinucleated giant cells?


- Plasma cells
- Macrophages
- Neutrophils
- None of the above

nolan x soju
41. In reversible cell injury all are true except
- ATP depletion is responsible for acute cellular swelling
- Can cause myocardial cells to cease contraction within 60 seconds
- ATP is generated anaerobically from creatine phosphate
- Mitochondrial swelling and degranulation of ER are hallmarks of irreversible cells injury
- Is associated with myelin figures

42. Which of the following pigments will accumulate as the result of peroxidation of cell
membranes?
- Bilirubin
- Lipochrome
- Melanin
- Hemomelanin
- Hemosiderin

43. In relation to atrophy, which of the following statements is FASLE?


- Decreased autophagic vacuoles
- Decreased myofilaments
- Decreased smooth endoplasmic reticulum
- None of the above

44. Transudation may occur due to the following EXCEPT


- Increased intravascular hydrostatic pressure
- Decreased intravascular oncotic pressure
- Lymphatic obstruction
- Inflammation

45. Initial rolling interactions in acute inflammation is mediated by which of the following?
- Interleukins
- Selectins - Integrins
- Immunoglobulins

46. Which of the following mediators most likely contributes to giant cell formation?
- Interferon ỿ
- Histamine
- IL – 1
- None of the above

47. What are the main opsonins involved in phagocytosis?


- Nitric oxide and histamine
nolan x soju
- IL-8 and leukotriene B4
- IgG and C3b
- Serotonin and C3b

48. Which of the following is a hallmark of cancer?


- Evasion of apoptosis
- Insensitivity to growth inhibitory signals
- Ability to metastasize
- All of the above
49. Which of the following statements is CORRECT in relation to dystrophic calcification?
- It rarely causes dysfunction
- It does not occur on heart valves
- It is formed by crystalline calcium phosphate
- It is specific to coagulative necrosis

50. Infarction
- The nuclei of malignant cells show
- Variability in size
- Variability to shape
- Hyperchromasia
- All of the above

51. In tumor invasion & dissemination


- Extracellular matrix cleavage products are growth promoting & chemotactic for tumor cells
- Tumor invasion through the extracellular matrix is by progressive expansile growth
- Tumor cells downregulates their adhesion molecules to reduce adhesion to each other
during vascular dissemination
- Organ selective homing of tumor cells are purely due to the anatomical location of the
primary tumor

52. Which of the following can be complication of thrombosis?


- Neoplastic transformation of the vessel wall
- Embolization
- Neovascularisation
- None of the above

53. Concerning the repair of a well opposed, clean surgical incision


- There is an initial inflammatory response
- 15% of original tissue strength is attained at one week
- Granulation tissue does not occur
- New collagen begins to accumulate after the first week nolan x soju
- Dermal appendages destroyed by the incision usually recover

54. All of the following vascular changes are observed in acute inflammation, except
- Vasodilation
- Stasis of blood
- Increased vascular permeability
- Decreased hydrostatic pressure

55. Which of the following chemical mediators act in chemotaxis during acute inflammation? -
Nitric oxide
- IL 12
- Serotonin
- None of the above

56. Which of the following terms refer to a malignant tumor of a smooth muscle?
- Lesomyoma
- Lesomyosarcoma
- Adenoma
- Rhabdomyosarcoma

57. Which of NOT evidence of irreversible cell injury?


- Acute cell swelling ( cellular edema )
- Nuclear pyknosis
- Rupture of the lysosomes
- All the above are evidence of irreversible injury

58. What is the outcome of necrosis?


- Calcification
- Repair
- Complete restitution regeneration
- All of the above

59. Which of the following is the best example of a pre-cancerous lesion?


- Endometrial atrophy leading to endometrial adenocarcinoma
- Endometrial hyperplasia leading to endometrial adenocarcinoma
- Thymic involution leading to lymphoma
- None of the above

60. Which of the following is an example of physiologic apoptosis?


- Involution of hormone-dependent tissue upon hormonal deprivation
nolan x soju
- Elimination of self-reactive lymphocytes
- Cell loss in the proliferating cell population
- All of the above
61. Irreversible cell injury is characterized by
- Dispersion of ribosomes
- Cell swelling
- Lysosomal rupture
- Cell membrane defects
- Nuclear chromatin clumping

62. Which of the following is true for hyperplasia?


- Bone marrow hyperplasia in high altitudes is an example of physiologic hyperplasia
- Hyperplasia is always pathological
- Hyperplasia is a decrease in the size of a tissue or an organ caused by deceased number of
constituent cells
- Hyperplasia is an example of an irreversible cell injury

63. Severe generalized edema is called


- Neoplasia
- Anasarca
- Pitting edema
- Myxoedema

64. The cellular accumulation in case of fatty change disorder of liver, may result from all of th
following factors EXCEPT
- Increased transport of precursor substances to cell
- Overproduction of accumulated material
- Decreased mobilization of accumulated material by cells
- Decreased utilization of accumulated material by cells
- Irreversible damage to mitochondria

65. All of the following phrases concerning Down’s syndrome are correct EXCEPT
- Most common chromosomal disorder
- Absence of Barr bodies in female patients
- Mental retardation
- Increased incidence of congenital heart disease and other congenital abnormalities
- Nondisjunction most frequent cause

66. A 4-year-old boy falls on a rusty nail and punctures his skin. The wound is cleaned and
covered with sterile gauze. Which of the following is the initial event in the healing process?
- Accumulation of acute inflammatory cells nolan x soju
- Deposition of proteoglycans and collagen
- Formation of a fibrin clot
- Macrophage-mediated phagocytosis of cellular debris

67. Which of the following terms refer to a malignant tumor of testicular epithelium?
- Seminoma
- Mesothelioma
- Hemangioma
- Lymphoma

68. Which of the following statements regarding tumorigenesis is TRUE?


- All malignant tumor are monoclonal
- Mutant alleles of proto-oncogene can incur cell transformation even in presence of their
normal counter parts (ie. It is a dominant phenomenon)
- Both mutant alleles of anti-oncogenes must exist for cell transformation to occur
- Phenotypic attributes of a tumor is acquired in a stepwise fashion - All of the above

69. Malignant tumor has the following characteristics


1. A woman who is allergic to cats visits a neighbor who has several cats. During the visit, she
inhales cat dander, and within minutes, she develops nasal congestion with abundant nasal
secretions. Which of the following substances is most likely to produce these findings? a.
Bradykinin
b. Complement C5a
c. Histamine
d. Interleukin-1 (IL-1)
e. Phospholipase C
f. Tumor necrosis factor (TNF)

2. A 35-year-old woman takes acetylsalicylic acid (aspirin) for arthritis. Although her joint pain
is reduced with this therapy, the inflammatory process continues. The aspirin therapy
alleviates her pain mainly through reduction in the synthesis of which of the following
mediators?
a. Complement C1q
b. Histamine
c. Leukotriene E4
d. Nitric oxide
e. Prostaglandins

3. A 19-year-old woman develops a sore throat and fever during the past day. Physical
examination shows pharyngeal erythema and swelling. Laboratory findings include nolan x soju
leukocytosis. She is given naproxen. Which of the following features of the acute
inflammatory response is most affected by this drug? a. Chemotaxis
b. Emigration
c. Leukocytosis
d. Phagocytosis
e. Vasodilation

4. In an experiment, neutrophils collected from peripheral blood are analyzed for a “burst” of
oxygen consumption. This respiratory burst is an essential step for which of the following
events in an acute inflammatory response? a. Attachment to endothelial cells
b. Generation of microbicidal activity
c. Increased production in bone marrow
d. Opsonization of bacteria Phagocytosis of bacteria
5. A 6-year-old child has a history of recurrent infections with pyogenic bacteria, including
Staphylococcus aureus and Streptococcus pneumoniae. The infections are accompanied by
a neutrophilic leukocytosis. Microscopic examination of a biopsy specimen obtained from
an area of soft tissue necrosis shows microbial organisms, but very few neutrophils. An
analysis of neutrophil function shows a defect in rolling. This child’s increased susceptibility
to infection is most likely caused by a defect involving which of the following molecules?
a. Complement C3b
b. Integrins
c. Leukotriene B4
d. NADPH oxidase
e. Selectins

6. A 36-year-old woman with pneumococcal pneumonia develops a right pleural effusion. The
pleural fluid displays a high specific gravity and contains large numbers of
polymorphonuclear (PMN) leukocytes. Which of the following best characterizes this pleural
effusion?
a. Fibrinous exudate
b. Lymphedema
c. Purulent exudate
d. Serosanguinous exudate
e. Transudate

7. A 68-year-old coal miner with a history of smoking and emphysema develops severe
airflow obstruction and expires. Autopsy reveals a “black lung,” with coal-dust nodules nolan x soju
scattered throughout the parenchyma and a central area of dense fibrosis. The coal
dust entrapped within this miner’s lung was sequestered primarily by which of the
following cells?
a. Endothelial cells
b. Fibroblasts
c. Lymphocytes
d. Macrophages
e. Plasma cells
8. A 25-year-old machinist is injured by a metal sliver in his left hand. Over the next few days,
the wounded area becomes reddened, tender, swollen, and feels warm to the touch.
Redness at the site of injury in this patient is caused primarily by which of the following
mechanisms? a. Hemorrhage
b. Hemostasis
c. Neutrophil margination
d. Vasoconstriction
e. Vasodilation

9. A 77-year-old woman experiences a sudden loss of consciousness, with loss of movement


on the right side of the body. Cerebral angiography shows an occlusion of the left middle
cerebral artery. Elaboration of which of the following mediators will be most beneficial in
preventing further ischemic injury to her cerebral cortex?
a. Bradykinin
b. Leukotriene E4
c. Nitric Oxide
d. Platelet-activating factor
e. Thromboxane A2

10. In an experiment, T lymphocytes from peripheral blood are placed in a medium that
preserves their function. The lymphocytes are activated by contact with antigen and
incubated for 4 hours. The supernatant fluid is collected and is found to contain a
substance that is a major stimulator of monocytes and macrophages. Which of the
following substances released into this fluid medium is most likely to stimulate
macrophages? a. Histamine
b. Interferon- γ
c. Leukotriene B4
d. Nitric Oxide
e. Phospholipase C
f. Tumor Necrosis Factor (TNF)

11. A 69-year-old woman has had transient ischemic attacks for the past 3 months. On physical
examination, she has an audible bruit on auscultation of the neck. A right carotid
nolan x soju
endarterectomy is performed. The curetted atheromatous plaque has a grossly yellow-tan,
firm appearance. Microscopically, which of the following materials can be found in
abundance in the form of crystals within cleftlike spaces? a. Cholesterol
b. Glycogen
c. Hemosiderin
d. Immunoglobulin
e. Lipofuscin

12. A screening chest radiograph of an asymptomatic 37-year-old man shows a 3- cm nodule in


the middle lobe of his right lung. The nodule is excised with a pulmonary wedge resection,
and sectioning shows a sharply circumscribed mass with a soft, white center. The serum
interferon gamma release assay is positive. Which of the following pathologic processes
has most likely occurred in this nodule? a. Apoptosis
b. Caseous necrosis
c. Coagulative necrosis
d. Fat necrosis
e. Fatty change
f. Gangrenous necrosis
g. Liquefactive necrosis

13. A 47-year-old man has a lung carcinoma with metastases. He receives chemotherapy. A
month later, histologic examination of a metastatic lesion shows many foci in which
individual tumor cells appear shrunken and deeply eosinophilic. Their nuclei exhibit
condensed aggregates of chromatin under the nuclear membrane. The pathologic process
affecting these shrunken tumor cells is most likely triggered by release of which of the
following substances into the cytosol? a. BCL2
b. Catalase
c. Cytochrome c
d. Lipofuscin
e. Phospholipase

14. A study of aging shows that senescent cells have accumulated damage from toxic
byproducts of metabolism. There is increased intracellular lipofuscin deposition. Prolonged
ingestion of which of the following substances is most likely to counteract this aging
mechanism? a. Antioxidants
b. Analgesics
c. Antimicrobials
d. Antineoplastic agents
e. Glucocorticoids

15. A 16-year-old boy sustained blunt trauma to his abdomen when he struck a bridge
nolan x soju
abutment at high speed while driving a motor vehicle. Peritoneal lavage shows a
hemoperitoneum, and at laparotomy, a small portion of the left lobe of the injured liver is
removed. Two months later, a CT scan of the abdomen shows that the liver has nearly
regained its size before the injury. Which of the following processes best explains this CT
scan finding? a. Apoptosis
b. Dysplasia
c. Hyperplasia
d. Hydropic change
e. Steatosis

16. An experimental study of steatohepatitis in metabolic syndrome reveals that hepatocyte


cell membrane injury with necrosis occurs in response to increased amounts of tumor
necrosis factor (TNF). When a pharmacologic agent inhibiting caspases is administered, cell
necrosis still occurs. Which of the following substances forms a supramolecular complex
that increases the generation of reactive oxygen species? a. Catalase Cytochrome c
b. Interleukin 1-beta converting enzyme
c. Receptor-interacting protein
d. Ubiquitin ligase

17. In an experiment, metabolically active cells are subjected to radiant energy in the form of
xrays. This results in cell injury caused by hydrolysis of water. Which of the following
intracellular enzymes helps to protect the cells from this type of injury?
a. Endonuclease
b. Glutathione peroxidase
c. Lactate dehydrogenase
d. Phospholipase Protease

18. A 72-year-old man died suddenly from congestive heart failure. At autopsy, his heart
weighed 580 g (normal 330 g) and showed marked left ventricular hypertrophy and
minimal coronary arterial atherosclerosis. A serum chemistry panel ordered before death
showed no abnormalities. Which of the following pathologic processes best accounts for
the changes of the aortic valve? a. Amyloidosis
b. Dystrophic calcification
c. Hemosiderosis
d. Hyaline change
e. Lipofuscin deposition
nolan x soju

1. A 30-year-old firefighter suffers extensive third-degree burns over his arms and hands.
This patient is at high-risk for developing which of the following complications of wound
healing?
a. Contracture
b. Dehiscence
c. Incisional hernia Keloid
d. Traumatic neuroma

2. A 29-year-old man with acute appendicitis undergoes surgical removal of the inflamed
appendix. The incision site is sutured. A trichrome-stained section representative of the
site with blue appearing collagen. How long after the surgery would this appearance most
likely be seen?
a. 1 day
b. 2 to 3 days
c. 4 to 5 days
d. 2 weeks 1 month

3. In an experiment, bacteria are inoculated into aliquots of normal human blood that have
been treated with an anticoagulant. It is observed that the bacteria are either
phagocytized by neutrophils or undergo lysis. Which of the following blood plasma
components is most likely to facilitate these effects?
a. Complement nolan x soju
b. Fibrin
c. Kallikrein
d. Plasmin
e. Thrombin

4. A 43-year-old man with a ventricular septal defect has had a cough and fever for the past
2 days. On examination, he has a temperature of 37.6° C and a cardiac murmur. A blood
culture grows Streptococcus, viridans group. His erythrocyte sedimentation rate (ESR) is
increased. Microbial cells are opsonized and cleared. Which of the following chemical
mediators is most important in producing these findings? a. Bradykinin
b. C-reactive protein
c. Interferon-γ
d. Nitric oxide Prostaglandin
e. Tumor necrosis factor (TNF)

5. A 28-year-old woman sustains a deep, open laceration over her right forearm in a
motorcycle accident. The wound is cleaned and sutured. Which of the following cell types
mediates contraction of the wound to facilitate healing? a. Endothelial cells
b. Fibroblasts
c. Macrophages
d. Myofibroblasts
e. Smooth muscle cells

6. A 77-year-old woman experiences a sudden loss of consciousness, with loss of movement


on the right side of the body. Cerebral angiography shows an occlusion of the left middle
cerebral artery. Elaboration of which of the following mediators will be most beneficial in
preventing further ischemic injury to her cerebral cortex? a. Bradykinin
b. Leukotriene E4
c. Nitric oxide
d. Platelet-activating factor
e. Thromboxane A2

7. A 7-year-old boy falls on a rusty nail and punctures his skin. The wound is cleaned and
covered with sterile gauze. Which of the following is the initial event in the healing
process?
a. Accumulation of acute inflammatory cells
b. Deposition of proteoglycans and collagen
c. Differentiation and migration of myofibroblasts
d. Formation of a fibrin clot
e. Macrophage-mediated phagocytosis of cellular debris nolan x soju

8. A woman who is allergic to cats visits a neighbor who has several cats. During the visit, she
inhales cat dander, and within minutes, she develops nasal congestion with abundant
nasal secretions. Which of the following substances is most likely to produce these
findings?
a. Bradykinin
b. Complement C5a
c. Histamine
d. Interleukin-1 (IL-1)
e. Phospholipase C
f. Tumor necrosis factor (TNF)
9. A 22-year-old woman nursing her newborn develops a tender erythematous area around
the nipple of her left breast. A thick, yellow fluid is observed to drain from an open fissure.
Examination of this breast fluid under the light microscope will most likely reveal an
abundance of which of the following inflammatory cells? a. Blymphocytes
b. Eosinophil
c. Mast cells
d. Neutrophils
e. Plasma cells

10. A 50-year-old man had chest pain persisting for 4 hours. A radiographic imaging procedure
showed an infarction involving a 4-cm area of the posterior left ventricular free wall.
Laboratory findings showed serum creatine kinase of 600 U/L. Which of the following
pathologic findings would most likely be seen in the left ventricular lesion 1 month later?
a. Chronic inflammation
b. Coagulative necrosis
c. Complete regeneration
d. Fibrous scar
e. Nodular regeneration

11. A 9-year-old boy receives a deep laceration over his right eyebrow playing ice hockey. The
wound is cleaned and sutured. Which of the following describes the principal function of
macrophages that are present in the wound 24 to 48 hours after the injury? a. Antibody
production
b. Deposition of collagen
c. Histamine release
d. Phagocytosis
e. Wound contraction

12. A 59-year-old woman tests positive for hepatitis A antibody. Her serum AST level is 275
nolan x soju
U/L, and ALT is 310 U/L. One month later, these enzyme levels have returned to normal.
Which phase of the cell cycle best describes the hepatocytes 1 month after her infection?
a. G0
b. G1
c. S
d. G2
e. M
13. A 29-year-old carpenter receives a traumatic laceration to her left arm. Which of the
following is the most important factor that determines whether this wound will heal by
primary or secondary intention? a. Apposition of edges
b. Depth of wound
c. Metabolic status
d. Skin site affected
e. Vascular supply

14. An 8-year-old girl with asthma presents with respiratory distress. She has a history of
allergies and upper respiratory tract infections. She also has history of wheezes associated
with exercise. Which of the following mediators of inflammation is the most powerful
stimulator of bronchoconstriction and vasoconstriction in this patient? a. Bradykinin
b. Complement proteins
c. Interleukin-1
d. Leukotrienes
e. Tumor necrosis factor-α

15. A 19-year-old woman develops a sore throat and fever during the past day. Physical
examination shows pharyngeal erythema and swelling. Laboratory findings include
leukocytosis. She is given naproxen. Which of the following features of the acute
inflammatory response is most affected by this drug? a. Chemotaxis
b. Emigration
c. Leukocytosis
d. Phagocytosis
e. Vasodilation

16. Which of the following preformed substances is released from mast cells and platelets,
resulting in increased vascular permeability in the lungs of the patient described in
previous question? a. Bradykinin
b. Hageman factor
c. Histamine nolan x soju
d. Leukotrienes (SRS-A)
e. Thromboxane A2

17. Patients with extensive endothelial injury from Escherichia coli sepsis have consumption of
coagulation factors as well as an extensive inflammatory response. Administration of
activated protein C is most likely to decrease this inflammatory response by reducing the
amount of which of the following substances? a. Complement
b. Fibrin
c. Kallikrein
d. Plasmin
e. Thrombin

18. In an experiment involving observations on wound healing, researchers noted that


intracytoplasmic cytoskeletal elements, including actin, interact with the extracellular
matrix to promote cell attachment and migration in wound healing. Which of the following
substances is most likely responsible for such interaction between the cytoskeleton and
the extracellular matrix?
a. Epidermal growth factor
b. Type III collagen
c. Integrin
d. Platelet-derived growth factor
e. Type IV collagen
f. Vascular endothelial growth factor

19. One month after an appendectomy, a 25-year-old woman palpates a small nodule beneath
the skin at the site of the healed right lower quadrant sutured incision. The nodule is
excised, and microscopic examination shows macrophages, collagen deposition, small
lymphocytes, and multinucleated giant cells. Polarizable, refractile material is seen in the
nodule. Which of the following complications of the surgery best accounts for these
findings?
a. Abscess formation
b. Chronic inflammation
c. Exuberant granulation tissue
d. Granuloma formation
e. Healing by second intention

20. A 45-year-old woman with autoimmune hemolytic anemia presents with increasing
fatigue. Which of the following mediators of inflammation is primarily responsible for
antibody-mediated hemolysis in this patient? a. Arachidonic acid metabolites
b. Coagulation proteins
c. Complement proteins nolan x soju
d. Kallikrein and kinins
e. Lysophospholipids
21. A 9-year-old boy has had a chronic cough and fever for the past month. A chest radiograph
shows enlargement of hilar lymph nodes and bilateral pulmonary nodular interstitial
infiltrates. A sputum sample contains acid- fast bacilli. A transbronchial biopsy specimen
shows granulomatous inflammation with epithelioid macrophages and Langhans giant
cells.Which of the following mediators is most likely to contribute to giant cell formation?
a. Complement C3b
b. Interferon-γ
c. Interleukin-1 (IL-1)
d. Leukotriene B4

22. Tum A 36-year-old woman with pneumococcal pneumonia develops a right pleural
effusion. The pleural fluid displays a high specific gravity and contains large numbers of
polymorphonuclear (PMN) leukocytes. Which of the following best characterizes this
pleural effusion?
a. Fibrinous exudate
b. Lymphedema
c. Purulent exudate
d. Serosanguineous exudate
e. Transudate

23. A 35-year-old woman takes acetylsalicylic acid (aspirin) for arthritis. Although her joint pain
is reduced with this therapy, the inflammatory process continues. The aspirin therapy
alleviates her pain mainly through reduction in the synthesis of which of the following
mediators?
a. Complement C1q
b. Histamine
c. Leukotriene E4
d. Nitric oxide
e. Prostaglandins

24. A 54-year-old man undergoes laparoscopic hernia repair. In spite of the small size of the
incisions, he has poor wound healing. Further history reveals that his usual diet has poor
nutritional value and is deficient in vitamin C. Synthesis of which of the following
extracellular matrix components is most affected by this deficiency? a. Collagen
b. Elastin
c. Fibronectin
d. Integrin
e. Laminin
nolan x soju

25. A 41-year-old man has had a severe headache for the past 2 days. On examination, his
temperature is 39.2° C. A lumbar puncture is performed, and the cerebrospinal fluid
obtained has a WBC count of 910/mm3 with 94% neutrophils and 6% lymphocytes. Which
of the following substances is the most likely mediator for the fever observed in this man?
a. Bradykinin
b. Histamine
c. Leukotriene B4
d. Nitric oxide
e. Tumor necrosis factor (TNF)
e. Anti–SS-A

5. A 2-year-old boy is found to have bilateral retinal tumors. Molecular studies demonstrate
a germline mutation in one allele of the Rb gene. Which of the following genetic events
best explains the mechanism of carcinogenesis in this patient? a. Balanced translocation
b. Expansion of trinucleotide repeat
c. Gene amplification
d. Loss of heterozygosity
e. Maternal nondisjunction

6. A 26-year-old woman has a history of frequent nosebleeds and increased menstrual


blood flow. On physical examination, petechiae and purpura are present on the skin of
her extremities. Laboratory studies show normal partial thromboplastin time (PTT),
prothrombin time (PT), and platelet count, but decreased von Willebrand factor activity.
This patient most likely has a derangement in which of the following steps in
hemostasis? a. Fibrin polymerization
b. Platelet Adhesion
c. Platelet aggregation
d. Prothrombin generation
e. Prothrombin inhibition
f. Vasoconstriction

7. Persons with sensitivity to body jewelry undergo skin patch testing with nickel nolan x soju
compounds. Within 24 to 48 hours there is focal erythema and induration of their skin,
accompanied by itching and pain. Histologically, there is an accumuation of neutrophils
and monocytes with some CD4 T cells. Which of the following cytokines produced by T
cells is most likely to be involved in mediating this reaction? a. IL-2
b. IL-5
c. IL-10
d. IL-12
e. IL-17
Which of the following types of necrosis is grossly opaque and chalky white
Fat necrosis

Which of the following types of necrosis is most commonly associated with ischemic injury:
Coagulative necrosis

3-5) For each example of necrosis listed on the left select the most suitable description from those
on the right.
Caseous necrosis – Cheese-like material
Coagulative necrosis – Dull, swollen firm area
Colliquative necrosis – Soft, liquefying material

A 28-year-old woman is pinned by falling debris during a hurricane. An x-ray film of the leg reveals
a compound fracture of the right tibia. The leg is immobilized in a cast for 6 weeks. When the cast
is removed, the patient notices that her right leg is weak and visibly smaller in circumference than
the left leg. Which of the following terms best describes this change in the patient's leg muscle?
Atrophy

Morphologic changes seen in chronic non-specific inflammation include an increase in:


Lymphocytes, plasma cells and fibrosis
nolan x soju

A 60-year-old man is rushed to the hospital with acute liver failure. He undergoes successful
orthotopic liver transplantation; however, the transplanted liver does not produce much
bile for the first 3 days. Poor graft function in this patient is thought to be the result of
“reperfusion injury.” Which of the following substances was the most likely cause of reperfusion
injury in this patient’s transplanted liver?
Reactive oxygen species

Which one of the following ultrastructural features is believed to allow for the increases
permeability of the vascular endothelium in acutely inflamed tissue? Gaps in
endothelial tight junctions

Which one of the following is not an endogenous mediator of increased vascular permeability?
Prostaglandin E2

A 28-year-old man with a history of radiation/bone marrow transplantation for leukemia presents
with severe diarrhea.
He subsequently develops septic shock and expires. Microscopic examination of the colon
epithelium at autopsy reveals numerous acidophilic bodies and small cells with pyknotic
Nuclei. Which of the following proteins most likely played a key role in triggering
radiationinduced cell death in this patient’s colonic mucosa?
P53

While in a home improvement center warehouse buying paint, a 35- year-old man hears "Look
out below!" and is then struck on the leg by a falling pallet rack, which strikes him on his left leg in
the region of his thigh. The skin is not broken. Within 2 days there is a 5 x 7 cm purple color to the
site of injury. Which of the following substances has most likely accumulated at the site of injury
to produce a yellow-brown color 16 days after the injury?
Hemosiderin

13-15) For each of the cell type listed on the left choose the most appropriate association from
those on the right

Alveolar macrophages -Phagocytic activity dependent on


oxygen

Kupffer cells – Lining cells of hepatic sinusoids

Langhans’ giant cells – Nuclei arranged peripherally in


the cytoplasm

nolan x soju
A 59-year-old woman had the loss of consciousness that persisted for over an hour. When she
becomes arousable, she cannot speak nor move her right arm or leg. A cerebral angiogram
revealed an occlusion to her left middle cerebral artery. Months later, a computed
tomographic (CT) scan shows a large 5 cm cystic area in her left parietal lobe cortex. This CT
finding is most likely the consequence of resolution from which of the following cellular
events? Liquefactive necrosis

The most characteristic feature of granulation tissue is the:


Growth of fibroblasts and new capillaries

The growth factor elaborated by macrophages, which recruits macrophages and fibroblasts to
wound site and induces all steps in angiogenesis is:
Fibroblast growth factor
Hyperplasia primarily operative in which of the following growth alterations Galactorrhea
in a c4 a prolactinoma

If the following events were places in their most probable order of occurrence following a
skin wound which would come first? Blood clot formation
Which of the following disorders is an example of metaplasia?
Squamous epithelium in the mainstem bronchus of a smoker

Which one of the following does not impair healing of a wound?


Good vascular supply

A diabetic patient has an ulcer on their great toe, which statement is false regarding its healing.
Large amounts of granulation tissue grow in from the periphery of the wound

Regarding hyperplasia which statement is correct


It is limited to cells capable of mitotic division

Which of the following is not a characteristic of irreversible cell injury?


Adaptable mitochondrial dysfunction

Which of the following pigments will accumulate as the result of peroxidation of cell membranes
Lipochrome

nolan x soju
If the following features were put in chronological order which would come fourth?
Production of mature plasma cells

An infiltrate of which one of the following cells is most characteristic of the early stages of acute
inflammation?
Neutrophils

Which one of the following is not an organ specific auto-immune disease? Rheumatoid
arthritis
A 45-year-old man has had a fever and dry cough for 3 days, and now has difficulty breathing and
a cough productive of sputum. On physical examination his temperature is 38.5 C. diffuse rales
are auscultated over lower lung fields. A chest radiograph shows a right pleural effusion. A right
thoracentesis is performed. The fluid obtained has a cloudy appearance with cell count showing
5500 leukocytes per microliter, 98% of which are neutrophils. Which of the following terms best
describes his pleural process
Purulent inflammation

Deposits of amyloid in whole organs can be diagnosed by histological staining with


Congo red
Which of the following are most frequent site of venous thrombosis
Veins of lower extremity

A 55-year-old woman has had discomfort and swelling of the left leg for the past week. On
physical examination, the leg is slightly difficult to move, but on palpation, there is no pain. A
venogram shows thrombosis of deep leg veins. Which of the following mechanisms is most likely
to cause this condition?
Immobilization

A 39 years old male developed a testicular mass which was removed and sent for pathological
examination, along with the additional studies. Which of the following is the most compelling
evidence that the lesion is malignant
Cells of the mass infiltrate a narrow band of tunica albuginea

Which of the following immunodeficiency states is most common?


Common variable immunodeficiency

Which of the following pair does not correctly match the tumor with its causative agent
Hepatocellular carcinoma – Hepatitis A virus nolan x soju

Cellular regeneration and restoration of normal morphology is possible in each of the


following conditions except? Diphtheric myocarditis

38-40) from the list on the right select the most appropriate definition for each of the terms on
the left.

Anaplasia – Complete loss of resemblance to tissue of origin


Differentiation – Degree of resemblance to tissue of origin

Dysplasia – Partial loss of resemblance to tissue of origin

For which of the following tumours is there a definite generic basis in a proportion of cases?
Colonic carcinoma

An important metabolic disturbance in shock is


A shift from aerobic to anaerobic metabolism
If a rare disorder with an early onset in life is inherited in such a way the male and female
offsprings are equally affected, only homozygous persons are affected, then the mode of
inheritance would be
Autosomal recessive

If eosinophils and mast cells are identified in a tissue biopsy from an inflammatory process, which
type of immune reaction is most likely occurring
Type 1

Primary causes of noninflammatory edema include all of the following except


Increased vascular permeability

An increased frequency of acute leukemia is most characteristic in patients with


Down syndrome

A 62-year-old woman has a 3-month history of weight loss, abdominal pain radiating to the back
and jaundice. The causal lesion show
Neoplasia
nolan x soju

Histological grading systems to classify prostatic carcinoma were evaluated by investigators


on two separate occasions. Results were compared between the investigators as well as
across occasions for each investigator. The purpose of the multiple measurements was to
Establish the reliability of the grading systems

Which is the guardian gene


P53 gene
An association between cigarette smoking and cancer includes all of the following except
Carcinoma of the thyroid

The gross and microscopic characteristics used to differentiate benign from malignant neoplasms
have been proved by testing them for correlation with
Behavior of neoplasms in human subjects

All of the following abnormalities may be seen in Marfan’s syndrome except


Sclerodactyly

Grading of cancer is based on which of the following statements


Degree of differentiation of tumor cells, anaplasia and number of mitosis

Review of a series of surgical pathology reports indicated that a certain neoplasm is grades as
grade 1 on a scale of 1 to 4. Clinically, some of the patients with this neoplasm are found to be
stage 1. Which of the following is the best interpretation of a neoplasm with stage 1 designation?
Is well-differentiated and localized

A 35-year-old woman had a firm nodule palpable on the dome of the uterus six years ago
recorded on routine physical examination. The nodule has slowly increased in size and now
appears to be about twice the size it was when first discovered. By ultrasound scan it is solid and
circumscribed. She remains asymptomatic. Which of the following neoplasms is she most likely to
have?
Leiomyoma

A study is performed to analyze characteristics of malignant neoplasms in biopsy specimens. The


biopsies were performed on patients who had palpable mass lesions on digital rectal examination.
Of the following microscopic findings, which is most likely to indicate that the neoplasm is
malignant?
Invasion
nolan x soju

A 45-year-old healthy woman has a routine check of her health status. She has no chest pain,
cough, or fever. A chest x-ray taken and shows a peripheral 2.5 cm diameter "coin lesion" in the
right mid-lung field. Which of the following biologic characteristics best distinguishes this lesion as
a neoplasm, rather than a granuloma
Uncontrolled ( autonomous ) growth
Alcohol keratin is composed of
Pre keratin

The cell most sensitive to ischemia is


Cortical neuron

One of the following is an apoptosis inhibitor gene


Bcl 2

A young man of 20, got a lacerated wound on his left arm, stitched-1 week later sutures were
remained-healing continued but the site became disfigured by prominent raised irregular nodular
scar, in next 2 months which of the following best describes the process :
Keloid formation

Bradykinin is a clevage product of


Kininogen

Chemotaxis is mediated by
Leukotriene B4 & C5a

Which leukotriene is adhesion factor for the neutrophil on the cell surface to attach to
endothelium
B4

Keloid scars are made up of


Dense collagen

In acute inflammation pain is mediated by


Bradykinin

Pale infarct are not seen in


Lung
nolan x soju

In shock characteristic feature is


Poor perfusion of tissue

AL type of amyloid is seen in


Multiple myeloma

Cancer suppressor gene is important in which malignancy


Retinoblastoma

Dystrophic calcification is most closely associated with


Necrosis

Granuloma formation is frequently associated with


A persistent irritant

Which one of the following is not associated with thrombosis?


Thrombocytopenia
Burkitt’s lymphoma – Epstein-Barr virus

Clear cell carcinoma of the vagina – Estrogen

Malignant melanoma – Ultraviolet light

A clinical study is performed of patients with pharyngeal infections. The most typical clinical
course averages 3 days from the time of onset until the patient sees the physician. Most of these
patients experience fever and chills. On physical examination, the most common findings include
swelling, erythema, and pharyngeal purulent exudate. Which of the following types of
inflammation did these patients most likely have?
Acute inflammation

Atrophy is most strikingly characterized by


A greater number of autophagic vacuoles

Dysplasia is characterized by all of the following EXCEPT


Relentless progression to malignant neoplasm
Fatty change with “tigroid” appearance in the myocardium is characteristic of nolan x soju
Chronic anemia

A 22-year-old man develops marked right lower quadrant abdominal pain over the past
day. On physical examination there is rebound tenderness on palpation over the right lower
quadrant. Laparoscopic surgery is performed, and the appendix is swollen, erythematous, and
partly covered by a yellowish exudate. It is removed, and a microscopic section shows infiltration
with numerous neutrophils. The pain experienced by this patient is predominantly the result of
which of the following two chemical mediators?
Prostaglandin and bradykinin

In von Hippel-Lindau disease, the multiple vascular lesions involving several organs are known as
Cavernous hemangiomas

Each type of necrosis is correctly paired with its most likely site of involvement EXCEPT
Liquefaction- spleen or lung

Tuberculous granuloma is characterized by:


Langhans giant cells

Which of the following is (are) most capable of hypertrophy?


Striated muscle cells of the skeletal muscles

All of the following abnormal clinical manifestations are correctly paired with the appropriate
compound or pigment deposited EXCEPT
Atrophy of hepatic and myocardial cells – lipofuscin

Which of the following types of necrosis is most often caused by sudden ischemia from vascular
occlusion Caseous necrosis

Which one of the following cellular changes is reversible?


Swelling of endoplasmic reticulum

Each classic term describing acute inflammation is correctly matched with its cause EXCEPT
Dolor – inflammatory cells infiltrating blood vessels
Deposits of amyloid in whole organs can be diagnosed by histological staining with Congo
red
nolan x soju

Klinefelter’s syndrome does not include which of the following conditions? Mental
retardation

The endogenous pigment (s) NOT derived from hemoglobin is (are) Lipofuscin

All of the following associations are correctly paired EXCEPT


XXXXX – Patau’s syndrome

A 40-year-old woman had bilateral silicone breast implants placed two years ago. Since that time,
she has noted increased firmness with slight deformity of the breast on the left. The implants are
removed, and there is evidence for leakage of the implant contents on the left. Which of the
following cell types is most likely to be most characteristic of the inflammatory response in this
situation?
Giant cell

Males with an XYY karyotype are usually susceptible to Severe


acne

Human immunodeficiency virus (HIV) has a special affinity for infecting which type of cell? CD4+
T cell

A 14-year-old healthy girl has a 0.3 cm reddish, slightly raised nodule on the skin of the upper part
of her chest found on a routine physical examination. She states that this lesion has been present
for years and has not appreciably changed in size or color. Which of the following neoplasms is
this nodule most likely to be?
Hemangioma

If a person has documented anemia but is asymptomatic, the physician can eliminate all of the
following causes EXCEPT
Mismatched blood unit
Which of the following findings provides for a definitive diagnosis of hemolytic anemia?
Red blood cell destruction

A deficiency of the red cell membrane component spectrin causes which of the following
conditions?
Hereditary spherocytosis nolan x soju

Which of the following coagulation factors is reduced or absent in classic hemophilia (hemophilia
A)?
VIII

Which of the following statements regarding heart failure is true?


Right-sided failure may result in hepatomegaly

All of the following statements regarding acute myocardial infarction are true EXCEPT
Histologic evidence is seen first at about 24 hours post infarction

Glomus tumors (glomangiomas) originate in structures that are responsible for which of
the following functions? Temperature regulation

The most serious complication of lower extremity thrombophlebitis is Pulmonary


infarction

“Cor bovinum” is a latin term that describes the appearance of a heart affected by which of the
1. A 62-year-old man is brought to the emergency room in a disoriented
state. Physical examination reveals jaundice, splenomegaly, and ascites.
Serum level is ALT, AST, alkali phosphatase and bilirubin are all elevated. A
liver biopsy demonstrates alcoholic hepatitis with mallory bodies. These
cytoplasmic structures are composed of interwoven bunnies of which of
the following proteins?

ANS: intermediate filament

2. A 52-year-old woman loses her right kidney following an automobile


accident. A CT scan of the abdomen 2 years later shows marked
enlargement of the left kidney. The renal enlargement is an example of
which of the following adaptations?

ANS: hypertrophy

3. A 60 year old man with chronic cystitis complains of urinary frequency


and pelvic discomfort. Digital rectal examination is unremarkable. Biopsy
of the bladder mucosa reveals foci of glandular epithelium and chronic
inflammatory cells. No cytologic signs of atypia or malignancy are
observed. Which of the following terms best describe the morphologic
response to the chronic injury in this patient?
nolan x soju
ANS: metaplasia

4. A 68 year old woman with a history of hyperlipidemia dies of cardiac


arrhythmia following a massive heart attack. Per oxidation of which of the
following molecules are primarily responsible for causing the loss fo
membrane integrity in cardiac myocytes in this patient?

ANS: phospholipids

5. A 56 year old women with a history of v and hypertension developed


progressive, right renal artery stenosis. Over time this patient’s right
kidney is likely to demonstrate which of the following morphologic
adaptation to a partial ischemia?

ANS: atrophy
6. A 40 year ld man is pulled from the ocean after a boating accident and
resuscitationed. Six hours later the patient develops acute renal failure.
Kidney biopsy reveals evidence of karyorrhexis and karyolysis in renal
tubular epithelial cells. Which of the following biochemical event
preceded these pathologic changes?

ANS: decreases in intracellular pH

7. A 24 year old woman with chronic depression ingests a bottle of


acetaminophen tablets. Two days later, she is jaundiced (elevated serum
bilirubin) and displays symptoms of encephalopathy, including
impairment in spatial perception. In the liver toxic metabolites of
acetaminophen are generated by which of the following organelles?

ANS: smooth endoplasmic reticulum

8. A 53 year old man has experienced severe chest pain for the past 6 hours.
On physical examination he is afebrile but has tachycardia. laboratory
studies show a serum troponin 1 of 10 ng/mL. A coronary angiogram is
performed emergently and reveals >90% occlusion of the left anterior
descending artery. In this setting, an irreversible injury to the myocardial
fibres will have occurred when which of the following cellular changes
occurs?

ANS: nuclei undergo karyorrhexis


nolan x soju
9. A 43 year old man has complained of mild burning substernal
painfollowing meals for the past 3 years. Upper GI endoscopy is
performed and biopsies are taken of an erythematous area of the lower
oesophageal mucosa 3 cm above the gastroesophageal junction. There is
no mass lesion, no ulceration, and no haemorrhage noted.the biopsies
show the presence of columnar epithelium with goblet cells. Which of the
following mucosal alteration is most likely represented by these findings?

ANS: metaplasia

10. An 80 year old man dies from complications of Alzheimer diseases.At


autopsy his heat is small (250g) and dark brown on sectioning.
microscopically, there is light brown perinuclear pigment with H&E
staining of the cardiac muscle fibres. Which of the following substance is
most likely increased in the myocardial fibres to produce this appearance
of his heart?

ANS: lipochrome from ‘wear and tear’

11. A 17 year old adolescent receives whole body radiation as part of


apreparatory regimen for bone marrow transplantation to treat acute
lymphocytic leukaemia. Which of the following cells and tissues in the
body is most likely to remain unaltered by the effects of this therapeutic
radiation?

ANS: neurons of cerebral cortex

12. A 55 year old man has a 30 year history of poorly controlleddiabetes


mellitus. He has had extensive black discolouration of skin and soft tissue
of his right foot with areas of yellowish exudate for the past 2 months.
staphylococcus aureus is cultured from this exudate. A below the knee
amputation is performed. The amputation specimen received in the
surgical pathology laboratory is most likely demonstrate which of the
following pathologic abnormalities?

ANS: Gangrene

13. Which of the following disorders is an example of coagulationnecrosis?


nolan x soju
ANS: Embolus to the superior mesenteric artery leading to bowel infraction

14. Malnutrition aging and ischaemia are most commonly associatedwith


which of the following:

ANS: atrophy

15. Cell injury in ischaemia is mostly caused by affecting the

ANS: mitcchondria

16. Liquefactive necrosis is most commonly seen in


ANS: the brain

17. Which of the following describes karyorrhexis

ANS: nucleus breaks up into smaller fragments scattered around the cytoplasm

18. Which of the following is true regarding atrophy?

ANS: it is an active process resulting in the shrinkage of cells

19.which of the following is not a characteristic of reversible cell injury?

ANS: cellular shrinkage

20. Chemically induced cell injury from carbon tetrachloride (ccl4)


andacetaminophen (tylenol) affect which organ?

ANS: liver

21. Fatty change with “tigroid” appearance in the myocardium ischaracteristic


of

ANS: chronic anemia

22. Chronic irritation (chemical or physical

ANS: metaplasia
nolan x soju
23. Increases demand, increased trophic stimulation

ANS: hyperplasia/ hypertrophy

24. Reduced oxygen supply, chemical injury, infection

ANS: injury/ necrosis/ apoptosis

25. Metabolic alterations, genetic or acquired

ANS: calcification/ intracellular accumulation

26. Decreased nutrients, stimulation


ANS: atrophy

27. A clinical study is performed of patients with pharyngeal infection. The


most typical clinical course averages 3 days from the time of onset until
the patient see the physician. Most of these patients experienced fever
and chills. On physical examination the most common finding is a
pharyngeal purulent exudate. Which of the following types of
inflammation did these patients most likely have?

ANS: acute inflammation

28. True statements concerning tissue repair by granulation tissueformation


include

ANS: a wound left open typically undergoes primary healing

29. Tuberculous granuloma is characterised by:

ANS: langhans giant cells

30. Each classic term describing acute inflammation is correctlymatched with


it’s cause except

ANS: dolor- inflammatory cells infiltrating blood

31. A 40 year old woman had bilateral silicone breast implants placedtwo
year ago. Since tat time she has noted increase firmness with slight
nolan x soju
deformity of the breast on the left. The implats are removed and there is
evidence for leakage of the implant contents not he left. Which of the
following cell types is most likely to be most characteristic of the
inflammatory response in this situation?

ANS: gains cell

32. Fluid that collects during acute inflammation and that has a
proteincontent exceeding 3 g/dl and a specific gravity exceeding 1.015 is
referred

ANS: an exudate
33. Once damaged a tissue or organ usually attempts to regenerateitself. All
of the following cell type are capable of regenerating tissues except

ANS: myocardial cells

A skin test with candida antigens shows 2 cm of induration at 48 hours.


The time of onset of the disease is best explained by

ANS: loss of passive immunity from the mother

41. A 2 year old boy healthy for the first six months of life. He then develops
recurrent infections. Serum concentration of complement, phagocytic function
and bactericidal function of neutrophils are normal.
A skin test with candida antigens shows 2 cm of induration at 48 hours. Which
of the following would most likely be abnormal?

ANS: serum concentration of IgG

42. TxA-2 is a characteristic product of

ANS: platelets

43. A granulomatous inflammatory response is diagnostic of:

ANS: tuberculosis

44. Examination of a lymph node reveals granulomas that show


nolan x soju
centralnecrosis. The finding is most consistent with a diagnosis of

ANS: cat scratch disease

45. Which of the following is most likely to be found


inhypogammaglobulinemia ( Burton disease )?

ANS: infections with homophiles influenzae

46. pulmonary involvement in systemic lupus erythematous ischaracteristized


by

ANS: diffuse interstitial fibrosis


47. The approaches for the prevention of allergic disease based on
itsmechanism of action include all of the following except

ANS: administration of antibiotics

48. Congo red with amyloid produces


Ans. Uncontrolled proliferation of the cells

11. All of the following are usual T cell functions except


Ans. Antigen presentation

12. The inflammatory mediators of cellular origin are


Ans. Histamine, serotonin, lyzozomal enzymes, leukotrienes, prostaglandins,
platelet activating factor.

13. Inflammation is caused by


Ans. Biological, physical, and chemical factors

14. Mast cells and blood basophils release Ans. Histamine, serotonin,
anaphilaron

15. Macrophages release


Ans. Interleukin-1

16. Mediators act on the cells through


nolan x soju
Ans. Surface receptors

17. ……
18. Which phase of inflammation is started with the reaction of
microcirculation
Ans. Phase of alteration

19. At the last stage of exudation is developed


Ans. Disturbance of blood reologic features

20. Mediators of endothelium-leukocyte adhesion are


Ans. ELAM-1, ICAM-1

21. During the acute inflammation within the first 6-24 hours are accumulated
preferably
Ans. Neutrophils
22. The adhesion molecules of endothelial cells are
Ans. ELAM-1, ICAM-1

23. Neutrofils (PMN cells) chemoattractant factors are Ans. Bacterial


products, C5a, Leukotriene B4

24. To exudative inflammation doesn’t belong Ans. Serous inflammation

25. Abscess is defined as


Ans. Focal supperative inflammation

26. Phlegmona is defined as


Ans. Diffuse suppurative inflammation

27. In the content of pus should not be Ans. Lymphocytes

28. Tuberculous granuloma is characterized by


Ans. Langerhans giant cells
nolan x soju
29. Tuberculous granuloma is characterized by
Ans. Caseous necrosis

30. Siphilitic granuloma is characterized by


Ans. Caseous necrosis

31. The surface adhesion molecule named E-selectin (also known as ELAM-1)
is located on
Ans. Endothelial cells

32. The complement component fragment which serves as a major opsonin is


Ans. C3b

33. All the chemical mediators listed increase vascular permeability except
Ans. IL-1

34. The enzyme myeloperoxidase is produced by


Ans. Neutrophils

35. All chemical reactants listed are oxygen dependent for bactericidal action
except
Ans. Lactoferina

36. The hallmark feature of granulomatous inflammation is Ans.


Epithelioid cell (activated macrophage)

37. The principal chemotactic chemical mediators are Ans. Leukotriene B4

38. Which of the following are needed for granuloma formation Ans. Cell
mediated immunity to the inciting agent

39. The vasoactive amines include


Ans. Serotonin, histamine

40. Human immunodeficiency virus (HIV) has a special affinity for infecting
which type of cell? nolan x soju

Ans. CD4+ T cell

41. Host-versus-graft reaction is not characterized by


Ans. A monoclonal B cell proliferation

42. Black carbon particles cleared from the blood by reticuloendothelial


system of a laboratory anomal, could be seen in cells in all of the following
organ components except
Ans. Glomeruli

43. All of the following statements describing leukocyte emigration from


vessels in area of inflammation are true except
Ans. Accompanying loss of red cells is passive

44. Once damaged, a tissue or organ normally attempts to regenerate itself.


All of the following cell types are capable of regenerating tissue except
64.Prostaglandin E has been postulated as participating in which of the
following regarding the process of inflammation?
Ans. Vasodilation

65.Which of the following cellular organelles is involved with phagocytosis of


bacteria and accumulation of cell debris?
Ans. Lysosome

66.Immunologic deficiencies may be observed in association with all of the


following except
Ans. Acute hepatic failure

67.Deposits of amyloid in whole organs can be diagnosed by histological


staining with
Ans. Congo red

68.Deposits of amyloid in whole organs can be diagnosed by staining the cut


surfaces with
Ans. Congo red nolan x soju

69.Which of the following proteins is most likely to be found in a vasculitis of


immune origin
Ans. Fibrin

70.Which of the following has no bearing on the vascular changes observed in


an anaphylactoid reaction?
Ans. Histones

71.Which of the following cell type is most crucial in chronic rejection


reaction?
Ans. Lymphocytes

72.The most specific test for systemic lupus erythematosus is the


demonstration of
Ans. Antibodies of double-stranded DNA

82. Systemic immune disease caused by formation of small soluble


antigenantibody aggregates within the circulation
Ans. Serum sickness

83. Prototype of cell-mediated response


Ans. Tuberculin reaction

84. This reaction is induced by intravenous injection of antibody followed by


subcutaneous antigenic challenge
Ans. Arthus reaction

85. Rapidly developing immunologic reaction resulting from the combination


of antigen and IgE
Ans. Anaphylaxis

86. .…………

87. Function as an opsonin in the inflammatory process Ans. C3b


nolan x soju
88. A polypeptide and an endogenous pyrogen produced by macrophages
Ans. Interleukin-1

89. Potent vasodilator


Ans. Prostaglandin PGI2

90. Chemotaxis and aggregation of neutrophils is related to Ans.


Leukotriene B4
91. Autoimmune pathogenic mechanisms seem to play a role in Ans.
Lupus nephritis
92. Reduced capacity for rejection of graft is encountered principally in
Ans. Radiation therapy
93. Increased permeability of venules during the acute inflammatory process
is not mediated by which of the following?
Ans. Prostaglandin E2

94. A granulomatous inflammatory response is diagnostic of


Ans. Tuberculosis

95. The epitheloid cells and mononucleated giant cells of granulomatous


inflammation are derived from
Ans. Monocyte-macrophages

96. Which of the following findings is an invariable histologic feature of


granulomatous inflammation ?
Ans. Multinucleated giant cells

97. The component of complement system that has biologic activity in


inflammation is
Ans. C3a

98. The antibody found in the sera of patients with systemic lupus
erythematosus include all following except
nolan x soju
Ans. Anti-basal membrane

99. When the components of the complement system interact, they do all
following except
Ans. Suppression of T cell activities

100. All the following statements regarding Human immunodeficiency


Virus (HIV) are true except
Ans. Shows selective tropism for T8 killer lymphocytes.

1. The complement component fragment which serves as a major


opsonin is
C3b
2. Each type of necrosis is correctly described except
Coagulation – loss of tissue architecture
3. All of the following factors cause cellular injury through the action of
free radicals except
Ischemia
4. Human immunodeficiency virus (HIV) has a special affinity for
infecting which type of cell?
CD4+ T cell
5. During the acute inflammation within the first 24 hours are
accumulated preferably
Neutrofils
6. The inflammatory mediators of cellular origin are nolan x soju
Histamine, serotonin, lizosomal enzymes, leukotrienes,
prostaglandins, platelet activating factor
7. Which of the following cellular changes is reversible?
Swelling of endoplasmic reticulum
8. Which of the following is (are) most capable of hypertrophy?
Striated muscle cells of the skeletal muscles
9. Fatty change with “tigroid” appearance in the myocardium is
characteristic of
Chronic anemia
10. The surface adhesion molecule named E-selectin (also known
as ELAM-1) is located on
Endothelial cells
Macrophage
72. Mediators act on the cells through
Surface receptors
73. Mediators of endothelium leukocyte adhesion are
ELAM-1, ICAM-1
74. Tuberculous granuloma is chracterized by
Caseous necrosis
75. Siphilitic granuloma is characterized by
Fibrinous necrosis
76. The unidirectional migration of leukocytes toward a target is
referred to as
Chemotaxis
77. Change of one kind mediator to another kind in the site of
inflammation causes
nolan x soju
Change of cellular forms in the site of inflammation
78. The term that describes the adherence of neutrophils and
monocytes to the vascular endothelium prior to movement
into the extravascular space is
Pavementing
79. Neutrofils (PMN cells) chemoattractant factors are
Bacterial products, C5a, Leukotriene B4
80. In the content of pus should not be
Giant cells
81. Which phase of inflammation is started with the reaction of
microcirculation
Release of cellular mediators
1. A 50-year-old chronic alcoholic presents to the emergency room with 12 hours of severe abdominal
pain, The radiates to the back and is associated with an urge to vomit. Physical examination discloses
exquisite abdominal tenderness. Laboratory studies show elevated serum amylase. Which of the
following morphologic changes expected in the peripancreatic tissue of this patient?

(A) Coagulative necrosis

(B) Caseous necrosis


(C) Fat necrosis

(0) Fibrinoid necrosis

(E) liquefactive necrosis

2. A 62-year-old man is brought to the emergency room in a disoriented state. Physical examination
reveals jaundice splenomegaly, and ascites. Serum levels of ALT, AST, alkaline phosphatase, and bilirubin
are all elevated. A liv demonstrates alcoholic hepatitis with Mallory bodies. These cytoplasmic structures
are composed of Interwoven bundles of which of the following proteins?

(A) as Antitrypsin

(B) D-Amyloid (AD)

(C) intermediate filaments

(D) Prion protein (PrP)

(e) a synuclein
nolan x soju

3. A 32-year-old woman with poorly controlled diabetes mellitus delivers a healthy boy at 38 weeks
of gestation. A of maternal hyperglycemia during pregnancy, pancreatic islets in the neonate would
be expected to show which following morphologic responses to injury?

(A) Atrophy

(B) Dysplasia

(C) Hyperplasia

(D) Metaplasia

(E) Necrosis

4. A 20-year-old man from China is evaluated for persistent cough, night sweats, low-grade fever, and
general A chest X-ray reveals findings "consistent with a Ghon complex.’’ Sputum cultures grow
acidfast bacilli, Exa of hilac lymph nodes In this patient would most likely demonstrate which of the
following pathologic changes?

(A) Caseous necrosis

(B) Coagulative necrosis

(C) Fat necrosis

(A) Cytoskeletal Intermediate filament loss

(B) Decreased intracellular pH from anaerobic gycolysis

(C) increased free radicals

(D) Mitochondrial swelling

(E) Nuclear chromatin clumping and decreased protein synthesis


(F) Reduced protein synthesis

20. A 43-year-old man has complained of mild burning substernal pain following meals for the past 3
years. Upper GI endoscopy is performed and biopsies are taken of an erythematous area of the lower
esophageal mucosa 3 cm above the gastroesophageal junction. There is no mass lesion, no ulceration,
and no hemorrhage noted. The biopsies show the presence of columnar epithelium with goblet cells.
Which of the following mucosal alterations is most likely represented by these findings?

(A) Dysplasia

(B) Hyperplasia

(C) Carcinoma

(D) Ischemia

(E) Metaplasia

21. A 59-year-old woman had the loss of consciousness that persisted for over an hour. When she nolan x soju
becomes arousable, she cannot speak nor move her right arm or leg. A cerebral angiogram revealed an
occlusion to her left middle cerebral artery. Months later, a computed tomographic (CT) scan shows a
large 5 cm cystic area in her left parietal lobe cortex. This CT finding is most likely the consequence of
resolution from which of the following cellular events?

(A) Liquefactive necrosis

(B) Atrophy

(C) Coagulative necrosis

(D) Caseous necrosis

(E) Apoptosis

22. An 80-year-old man dies from complications of Alzheimer disease. At autopsy, his heart is small
(250 gm) and dark brown on sectioning. Microscopically, there is light brown perinuclear pigment with
H&E staining of the cardiac muscle fibers. Which of the following substances is most likely increased in
the myocardial fibers to produce this appearance of his heart?

(A) Hemosiderin from iron overload

(B) Lipochrome from ‘wear and tear’

(C) Glycogen from a storage disease

(D) Cholesterol from atherosclerosis

(E) Calcium deposition following necrosis

29. Which of the following disorders is an example of metaplasia?

(A) Increased Goblet cells in mainstem bronchus of a smoker

(B) Squamous epithelium in the mainstem bronchus of a smoker


(C) Proliferative endometrial glands in a woman on unopossed estrogen

(D) Hyperkeratosis of the skin in a pt with Psoriasis

(E) Multinucleated giant cells in a granuloma

30. Malnutrition, Aging and ischaemia are most commonly associated with which of the following

(A) Hypertrophy

(B) Atrophy

(C) Hyperplasia (D) Dysplasia

31. Which of the following morphological changes is NOT associated with coagulative necrosis?

A) Karyorrhexis
nolan x soju
(B) Abscess

(C) Pyknosis

(D) Karyolysis

32. Cell Injury in ischaemia is mostly caused by affecting the

(A) Lysosomes

(B) Nucleus

(C) Mitochondria

(D) Plasma membrane

33. Which of the following is a feature of necrosis but NOT apoptosis?

(A) Phagocytosis by neighbouring cells (B)

Cell shrinkage/atrophy

(C) Inflammation of neighbouring cells

(D) Fragmentation of cell

34. Liquefactive necrosis is most commonly seen in

(A) The kidneys

(B) The feet

(C) The brain

(D) The heart

35. Which type of necrosis is seen in myocardial infarctions?

(A) Colliquative
(B) Gangrenous

(C) Caseous

(D) Coagulative

36. Which of the following describes karyorrhexis?

(A) Excess fluid is taken into the cell, resulting in a turgid appearance

(B) The nucleus is extruded from the cell

(C) The nucleus becomes smaller and stains deeply basophilic

(D) Nucleus breaks up into smaller fragments scattered around the cytoplasm

37. Which type of necrosis is commonly associated with tuberculosis?


nolan x soju
(A) Caseous

(B) Gangrenous

(C) Fat

(D) Colliquative

38. Which of the following is true regarding atrophy?

(A) It is an active process resulting in the shrinkage if cells

(B) It is a passive process where the number of cells in an organ reduces

(C) it is a type of reversible cell death

(D) It is a type of irreversible cell death

39. Regarding hyperplasia which statement is correct

(A) It is never seen in the same tissue as hypertrophy

(B) it is seen in cardiac muscle in hypoxic patients

(C) It is limited to cells capable of mitotic division

(D) It is rarely physiologic

(E) complete removal of excess hormone triggers will slow progression but not reverse hyperplastic
changes

40. Which of the following is NOT characteristic of reversible cell injury?

(A) Reduced oxidative phosphorylation

(B) ATP depletion

(C) Cellular shrinking


(D) Changes in ion concentrations

(E) Water influx

41. Which of the following is NOT a characteristic of irreversible cell injury?

(A) Necrosis, which is always pathologic or apoptosis

(B) Intracellular accumulations (e.g. lipids) or calcification

(C) Structural changes (e.g. mitochondrial densities)

(D) Profound disturbances in membrane function

(E) Adaptable mitochondrial dysfunction

42. Chemically Induced cell Injury from carbon tetrachloride (CC4) and acetaminophen (Tylenol) affect
which organ?
nolan x soju
(A) Brain

(B) Kidneys

(C) Pancreas

(D) Spleen (E) Liver

43. Which of the following pigments will accumulate as the result of peroxidation of cell membranes?

(A) bilirubin

(B) lipochrome

(C) melanin

(D) hemomelanin

(E) hemosiderin

44. Fatty change with "tigroid" appearance in the myocardium is characteristic of

(A) Alcoholic cardiac injury


PATHOLOGY EXAM
Test Option 20122
DIRECTIONS: Each of the numbered items or incomplete statements in this section is followed by answers or by completions of the
statement. Select the -ONE lettered answer or completion that is BEST in each case and fill in the circle containing the corresponding letter
on the answer sheet

1. All of the following vascular changes are observed in acute inflammation, except: (A) Vasodilation
(B) Stasis of blood.
(C) Increased vascular permeability.
(D) Decreased hydrostatic pressure.Answer is D

2. Grandma falls down the steps leading to the entrance of the house of a relative hosting a family reunion,
who is heard to remark, "I've been meaning to get that loose step fixed." Grandma is hospitalized for
surgery to replace the broken hip she sustains and is then moved to a nursing home, but she is unable to
ambulate until about a month later, when she dies suddenly. Which of the following is most likely to be the
immediate cause of death found at autopsy?
(A) Squamous cell carcinoma of lung
(B) Tuberculosis
(C) Pulmonary embolism
(D) Pneumonia with pneumococcus
(E) Congestive heart failure (C) CORRECT.

3. Keloid scars are made up of


(A) Dense collagen
(B). Loose fibrous tissue
(C). Granulomatous tissue
(D). Loose areolar tissue

Answer is A

4. Each of the following is a characteristic of the plaques seen in multiple sclerosis EXCEPT (A) early
nolan x soju
complete' loss of axons
(B) gliosis
(Q loss of oligodendroglia
(D) mononuclear inflammatory infiltrate
(E) preferential paraventricular distributionAnswer is A

5. During the first two months of life, the microorganism most frequently responsible for meningitis is
(A) Escherichia coli
(B) Hemophilus influenzae
(C) Neisseria meningitidis
(D) Staphylococcus
aureus
(E) Streptococcus pneumonia
Answer is A

6. A study of pathologic findings in the islets of Langerhans is performed. It is observed that insulitis may occur in the islets. The
inflammatory infiltrate is predominantly composed of T lymphocytes. Which of the following is most likely to occur as a consequence of
insulitis? (A) Neoplasia
(B) Malabsorption
(C) Ketoacidosis
(D) Obesity
(E) Sepsis
C is correct

7. A 20-year-old man incurs blunt trauma to his upper outer arm. On physical examination, there is a 2 x 3 cm contusion. The initial soft
tissue bleeding stops in a few minutes and the size of the bruise does not increase. Which of the following chemical mediators is most
important in this episode of hemostasis?

1
(A) Leukotriene
(B) Prostaglandin
(C) Prostacyclin
(D) Plasminogen
(E) Thromboxane
(E) CORRECT.

8. A subdural hematoma is most likely to be associated with a traumatic tear of the (A)
bridging cerebral veins
(B) middle cerebral artery
(C) middle meningeal artery
(D) superficial temporal artery
(E) superior longitudinal sinus
Answer is A

9. A 60-year-old man presents with dizziness, nausea, and


severe shortness of breath of several months’ duration. Physical
examination shows hepatomegaly, ascites, and anasarca. His
blood pressure is 200/115 mm Hg. An X-ray fi lm of the chest
demonstrates cardiomegaly and mild pulmonary edema.
Although different mechanisms may have contributed to the
pathogenesis of hypertension in this patient, the common end
result for all of them is which of the following?
(A) Arterial cystic medial necrosis
(B) Decreased plasma oncotic pressure
(C) Generalized vasodilation
(D) Increased peripheral vascular resistance
(E) Increased vascular permeability

Answer is D

10. A 68-year-old woman has survived multiple episodes of pulmonary thromboembolism during the past three months. Which of the
following is the most likely underlying condition leading to this patient's recurrent pulmonary thromboembolism? (A) Micronodular
cirrhosis of the liver
(B) Adenocarcinoma of the pancreas
(C) Thrombocytopenia
(D) Familial hypercholesterolemia nolan x soju
(E) Mitral valve endocarditis(F) Type II diabetes mellitus (B) CORRECT.

11. A 44-year-old African-American man has had elevated blood pressure for years which has not been treated. He now has severe
headaches. On physical examination his blood pressure is 275/150 mm Hg. Laboratory studies show Hgb 13.8 g/dL, serum glucose 76
mg/dL, and creatinine 3.5 mg/dL. These findings are most likely to be associated with which of the following pathologic lesions involving
his kidneys?
(A) Hyperplastic arteriolosclerosis
(B) Hyaline arteriolosclerosis
(C) Monckeberg medial calcific sclerosis
(D) Atherosclerosis
(E) Thrombophlebitis
(A) CORRECT.

12. Which of the following' skin lesions is considered premalignant?


(A) Fibrous histiocytoma
(B) Lichen planus
(C) Psoriasis
(D) Seborrheic keratosis
(E) Solar (actinic) keratosis
Answer is E

13. A study is performed involving persons who have a history of diabetes mellitus type 1 or type 2. These patients are found to have
cellular injury that results from glycosylation end products and from sorbitol accumulation within cells. The same patients are also
shown to have ischemic tissue damage from accelerated and advanced atherosclerosis. Which of the following complications is most
likely to result from atherosclerosis in these patients?

2
(A) Chronic renal failure
(B) Impotence
(C) Stroke
(D) Cataracts
(E) Retinopathy
C is CORRECT.

14. A 45-year-old woman who has been bedridden for several weeks has the onset of left sided chest pain along with dyspnea. She has
some tenderness in her left leg, which has a slightly greater circumference in the thigh than the right. A ventilation/perfusion scan shows
evidence for a left lower lobe perfusion defect. Which of the following vascular diseases is most likely to cause these findings: (A) Hyaline
arteriolosclerosis
(B) Monckeberg's medial calcific sclerosis
(C) Complex calcified coronary atherosclerosis
(D) Arterial mural thrombosis(E) Phlebothrombosis
E is CORRECT.

15. Some cells demonstrate glucose uptake regardless of the plasma insulin level. In a person who has had persistent hyperglycemia for
years, cellular injury can occur. Which of the following cell types is most likely to show injury from hyperglycemia: (A) Cardiac muscle
cells
(B) Fibroblasts
(C) Steatocytes
(D) Neurons
(E) Smooth muscle cells.
(D) CORRECT.

16. During hospitalization, a 40-year-old woman develops thrombophlebitis. She recovers and is discharged. She returns to her job as an
electrician. A couple of months later, which of the following terms would best describe the process seen in a femoral vein after recovery
from her thrombophlebitis:
(A) Acute inflammation
(B) Rupture
(C) Organization
(D) Embolization(E) Propagation
C is CORRECT.

17. The most common cause of viral pneumonia in infants under 6 months is
(A) ECHO virus
(B) influenza virus, group A
(C) rhinovirus
(D) respiratory syncytial virus
(E) coronavirus nolan x soju
Answer is D

18. Fatty change with “tigroid” appearance in the myocardium is characteristic of (A) Alcoholic cardiac
injury
(B) Diabetes mellitus
(C) Chronic anemia
(D) Hyperthyroidism
(E) Obesity
The answer is C

19. A 32-year-old Rh-negative woman was delivered of her third child. She had not sought medical care during pregnancy. Upon
admission,, her saline agglutinin titer for Rh+ cells was 1:4, and indirect antiglobulin (Coombs) test titer was 1:32. The infant had a
hemoglobin of 10 g/dL and became severely jaundiced, as her second child had been. The greatest hazard for the infant is (A)
cardiac failure secondary to anemia
(B) hypoglycemia secondary to pancreatic islet hyperplasia
(C) jaundice associated with an increased concentration of conjugated (direct) bilirubin
(D) kernicterus associated with an increased concentration of unconjugated (indirect) bilirubin(E) renal failure secondary to
hemolysis
Answer is D

20. The most important prognostic factor for human cancer is


(A) Tumor grade

3
(B) Tumor stage
(C) Lymphocytic infiltration
(D) Vascular invasion
(E) The mitotic index
The answer is B
21. A 52-year-old man has an ulcerated area on the sole of his foot that has not healed for 2 months. He is 180 cm tall, weighs 126 kg, and
has continued to gain weight gradually. He has not had any major illnesses. His blood pressure is normal. Which of the following
laboratory tests performed on serum from a blood sample would be most useful in elucidating the underlying cause for his problem: (A)
Antithrombin III
(B) Cortisol
(C) Creatine kinase
(D) Glucose
(E) Carcinoembryonic antigen
D is CORRECT.

22. Severe cell injury due to ischemia is characterized by


(A) increased cellular capacity for oxidative phosphorylation
(B) a loss of intracellular calcium
(C) shrinkage of mitochondria(D) decreased intracellular pH
(E) decreased intracellular lactate
Answer is D

23. Coagulation necrosis is most often a result of


(A) digestion by lipolytic and proteolytic enzymes
(B) cellular damage caused by chemical poisons
(C) postmortem autolysis
(D) infection by mycobacteria (M. tuberculosis)
(E) local hypoxia caused by loss of blood supply Answer is E

24. Which of the following pigments will accumulate as the result of peroxidation of cell membranes?
(A) Bilirubin
(B) Lipochrome
(C) Melanin
(D) Hemozoin
(E) Hemosiderin
Answer is B

25. Cellular regeneration and restoration of normal morphology is possible in each of the following conditions EXCEPT (A)
diphtheritic myocarditis
(B) damage to proximal convoluted renal tubules by mercuric salts nolan x soju
(C) acute erosion of gastric mucosa
(D) viral hepatitis A
(E) squamous metaplasia of the bronchial epithelium
Answer is A

26. A 57-year-old man has had blood pressure measurements in the range of 160/95 to 180/110 mm Hg for many years. He has
taken no medications. A renal scan reveals kidneys of normal size for age. These findings with benign nephrosclerosis are most likely to
occur with which of the following vascular changes:
(A) Hyaline arteriolosclerosis
(B) Monckeberg's medial calcific sclerosis
(C) Complex calcified atherosclerosis
(D) Arterial mural thrombosis
(E) Hyperplastic arteriolosclerosis
A is CORRECT.

27. A deficiency of the red cell membrane component spectrin causes which of the following conditions?
(A) Pernicious anemia
(B) Hereditary spherocytosis
(C) Sickle cell anemia
(D) Thalassemia major
(E) Blackfan-Diamond syndrome

4
The answer is B

28. The pathogenesis of granulomatous reactions includes each of the following EXCEPT
(A) activated macrophages
(B) antibodies
(C) bacterial lipids
(D) insoluble materials
(E) T lymphocyte
Answer is B
29. Development of tensile strength in a healing wound parallels the
(A) appearance of mucopolysaccharides
(B) ingrowth of fibroblasts
(C) appearance of reticulin fibers
(D) appearance of collagen
(E) contraction of the scar
Answer is D

30. The gross and microscopic characteristics used to differentiate benign from malignant neoplasms have been proved by testing them
for correlation with
(A) electron microscopic features of the neoplasms
(B) chemical analyses of neoplasms for DNA content
(C) analyses of chromosome number and configuration
(D) growth of neoplastic cells in tissue culture
(E) behavior of neoplasms in human subjectsAnswer is E

31. In which of the following conditions bilateral contracted kidneys are characteristically seen?
(A). Amyloidosis.
(B). Diabetes mellitus.
(C) . Rapidly progressive (crescentic) glomerulonephritis.
(D). Benign nephrosclerosis.
Answer is D

32. Which one of the following diseases characteristically causes fatty change in liver?
(A) Hepatitis B virus infection.
(B). Wilson’s disease.
(C). Chronic alcoholism.
(D) Hepatitis C virus infection.
Answer is C

33. Splenectomy increases the risk for


(A) Hodgkin disease
(B) idiopathic thrombocytopenic purpura
(C) myeloid metaplasia
(D) pneumococcal infection nolan x soju
(E) polycythemia vera
Answer is D

34. Endothelium, leukocyte interaction during inflammation mediatedthrough


(A). Selectin
(B). Vimentin
(C). Defensin
(D). Endothelin

Answer is A

35. Factors that contribute to the pathogenesis of amyloidosis include each of the following
EXCEPT(A) chronic infection
(B) deficiency of amylase
(C) inherited susceptibility
(D) malignant plasma cells
(E) rheumatoid arthritisAnswer is B

5
36. Which of the following is most likely to be found in hypogammaglobulinemia (Bruton disease)?
(A) A familial history of similar diseases in females
(B) Thymic agenesis
(C) Infections with Hemophilus influenzae
(D) Inflammatory bowel disease
(E) Leukopenia
Answer is C
37. A 58-year-old man is hospitalized for evaluation of recent intermittent upper abdominal pain. History and physical examination
reveal a 15-lb weight loss over recent months and upper abdominal tenderness without evidence of a mass or ascites. The best diagnostic
approach and most likely preliminary diagnosis would be
(A) Oral cholecystography for the diagnosis of chronic cholecystitis with cholelithiasis
(B) Serum bilirubin measurement for the diagnosis of chronic cholecystitis with cholelithiasis
(C) Endoscopy for the diagnosis of carcinoma of the ampulla of Vater
(D) Celiotomy for the diagnosis of carcinoma of the head of the pancreas
(E) Computed tomography for the diagnosis of carcinoma of the body of the pancreas
The answer is E

38. Complement is involved in the pathogenesis of each of the following EXCEPT


(A) Arthus reaction
(B) contact dermatitis
(C) erythroblastosis fetalis
(D) hemolytic transfusion ABO reaction
(E) serum sickness
Answer is B

39. Atherosclerotic plaques are LEAST likely to contain


(A) collagen
(B) cholesterol
(C) macrophages
(D) neutrophils
(E) smooth muscle cellsAnswer is D

40. What is the name given to the hyaline masses often seen in the cytoplasm of hepatocytes in patients with alcoholic hepatitis?
(A) Councilman bodies
(B) Negri bodies
(C) Mallory’s bodies
(D)Michaelis-Gutmann bodies
(E) Rotor’s bodies
The answer is C

41. A 52-year-old recent immigrant from Vietnam complains of abdominal swelling, weight loss, and
upper abdominal pain of 3 weeks in duration. His past medical history includes malaria and infection nolan x soju
with the liver fl uke Clonorchis sinensis. The liver is hard to palpation. An abdominal CT scan shows a
hypoattenuated mass with lobulated margins in the liver. A biopsy discloses well-differentiated
neoplastic glands embedded in a dense fi brous stroma. Which of the following is the most likely
diagnosis?
(A) Carcinoma of the gallbladder
(B) Cholangiocarcinoma
(C) Hemangiosarcoma
(D) Hepatocellular carcinoma
(E) Metastatic colon adenocarcinoma

Answer is B

42. An adolescent presents with lassitude, jaundice, fever, hypersplenism, and Kayser-Fleischer rings. Which of the
following disorders is most likely?
(A) Alpha1- antitrypsin deficiency
(B) Wilson’s disease
(C) Hepatitis B infection
(D) Gilbert’s syndrome
(E) Idiopathic hemochromatosis

6
The answer is B

43. Males with an XYY karyotype are usually susceptible to


(A) depression
(B) growth retardation
(C) palpitations
(D) priapism
(E) severe acne
Answer is E

44. All of the following clinical features are likely to be found in a patient with nephrotic syndrome
EXCEPT(A) Proteinuria
(B) Hypoalbuminemia
(C) Red cell casts
(D) Hyperlipidemia
(E) Edema
The answer is C

45. An increased frequency of acute leukemia is most characteristic in patients with(A) cystic
fibrosis
(B) Down syndrome
(C) Klinefelter syndrome
(D) Tay-Sachs disease
(E) Turner syndromeAnswer is B

46. Which of the following statements best characterizes membranous glomerulopathy?


(A) It is the most common cause of nephritic syndrome in children
(B) Affected patients usually present with acute renal failure
(C) The characteristic pathology is best described as “diffuse proliferative glomerulonephritis”
(D) Electron microscopy demonstrates numerous subepithelial immune-type deposits(E) It usually is responsive to steroid
therapy
The answer is D

47. All of the following statements regarding postinfectious glomerulonephritis are true EXCEPT
(A) The disease follows infection with group A -hemolytic streptococci
(B) Large subepithelial immune-type deposits are seen by electron microscopy
(C) The histologic picture is that of diffuse proliferative glomerulonephritis
(D) The clinical picture is characteristic of acute nephritis
(E) Most affected children progress to chronic renal failure
The answer is E

48. A 58-year-old woman is brought to the emergency department4 hours after vomiting blood and
experiencing bloody stools. The patient was diagnosed with alcoholic cirrhosis 2 years ago.
Endoscopy reveals large esophageal varices, one of which is actively bleeding. Which of the nolan x soju
following best explains the pathogenesis of dilated esophageal veins in this patient?
(A) Decreased intravascular oncotic pressure
(B) Increased capillary permeability
(C) Increased intravascular hydrostatic pressure
(D) Vasoconstriction of arterioles
(E) Vasodilatation of capillaries

The answer is C

49. A kidney biopsy specimen that shows subendothelial granular electron-dense deposits is characteristic of which of
the following disease states?
(A) Rapidly progressive glomerulonephritis
(B) Poststreptococcal glomerulonephritis
(C) Membranous glomerulonephritis
(D) Systemic lupus erythematosus
(E) Goodpasture’s syndrome
The answer is D

7
50. Cor pulmonale is a result of
(A) a healed myocardial infarct
(B) aortic stenosis
(C) emphysema
(D) rheumatic fever
(E) systemic hypertensionAnswer is C

51. In an adult patient, which of the following diseases would most frequently be associated with a smaller than normal spleen? (A)
Myelofibrosis
(B) Sickle cell anemia
(C) Infectious mononucleosis
(D) Chronic myelocytic leukemia
(E) Thalassemia major
Answer is B

52. A clinical study is performed of patients with pharyngeal infections. The most typical clinical course averages 3 days from the time of
onset until the patient sees the physician. Most of these patients experienced fever and chills. On physical examination, the most
common finding is a pharyngeal purulent exudate. Which of the following types of inflammation did these patients most likely have?
(A) Granulomatous inflammation
(B) Acute inflammation
(C) Abscess formation
(D) Resolution of inflammation
(E) Chronic inflammation

The answer is B
53. A 45-year-old man has had a fever and dry cough for 3 days, and now has difficulty breathing and a cough productive of sputum. On
physical examination his temperature is 38.5 C. Diffuse rales are auscultated over lower lung fields. A chest radiograph shows a right
pleural effusion. A right thoracentesis is performed. The fluid obtained has a cloudy appearance with a cell count showing 5500
leukocytes per microliter, 98% of which are neutrophils. Which of the following terms best describes his pleural process? (A) Serous
inflammation
(B) Purulent inflammation
(C) Fibrinous inflammation
(D) Chronic inflammation
(E) Granulomatous inflammation
The answer is B

54. All of the following associations are correctly paired EXCEPT


(A) Fragile X – mental retardation
(B) XXY – male infertility
(C) XXXXX – Patau’s syndrome
(D) 5p – characteristic cry
The answer is C

55. A 40-year-old woman had bilateral silicone breast implants placed two years ago. Since that time, she has noted increased firmness
with slight deformity of the breast on the left. The implants are removed, and there is evidence for leakage of the implant contents on
the left. Which of the following cell types is most likely to be most characteristic of the inflammatory response in nolanthis x soju
situation? (A) mast cell
(B) Eosinophil(C) Neutrophil
(D) Plasma cell
(E) Giant cell
The answer is E

56. The histologic finding most characteristic of chronic bronchitis is


(A) atrophy of smooth muscle
(B) degeneration of ganglion cells
(C) enlargement of mucous glands
(D) metaplasia of cartilage
(E) thickening of the basement membrane

Answer is C
57. Pulmonary infarcts are characterized by each of the following EXCEPT
(A) subpleural location
8
(B) increased frequency in patients with heart failure
(C) pale color
(D) wedge shape
(E) becoming abscesses when infected

Answer is C
58. Surgical therapy is LEAST likely to cure
(A) bronchogenic adenocarcinoma
(B) carcinoid
(C) hamartoma
(D) small-cell (oat-cell) carcinoma
(E) squamous cell carcinoma

9
14

Pathology Exam
Test option - 20124

Directions: Each question below contains five suggested answers. Choose the one best response to each question.

1. A clinical study is performed of patients with pharyngeal infections. The most typical clinical course averages 3 days
from the time of onset until the patient sees the physician. Most of these patients experienced fever and chills. On
physical examination, the most common finding is a pharyngeal purulent exudate. Which of the following types of
inflammation did these patients most likely have?
A. Granulomatous inflammation
B. Abscess formation
C. Resolution of inflammation
D. Acute inflammation
E. Chronic inflammation

The answer is D

2. Atrophy is most strikingly characterized by


A. Fewer myofilaments
B. A greater number of autophagic vacuoles
C. Fewer mitochondria
D. Less smooth endoplasmic reticulum
E. Less rough endoplasmic reticulum The answer is B

3. Dysplasia is characterized by all of the following EXCEPT


A. Relentless progression to malignant neoplasm
B. Disorderly proliferation of cells
C. Pleomorphism nolan x soju
D. Hyperchromaticity
E. Most frequently occurs in epithelium

The answer is A

4. Fatty change with “tigroid” appearance in the myocardium is characteristic of


A. Alcoholic cardiac injury
B. Chronic anemia
C. Diabetes mellitus
D. Hyperthyroidism
E. Obesity

The answer is B

a. A 22-year-old man develops marked right lower quadrant abdominal pain over the past day. On physical examination
there is rebound tenderness on palpation over the right lower quadrant. Laparoscopic surgery is performed, and the
appendix is swollen, erythematous, and partly covered by a yellowish exudate. It is removed, and a microscopic
section shows infiltration with numerous neutrophils. The pain experienced by this patient is predominantly the result
of the formation of which of the following two chemical mediators?
A. Complement C3b and lgG
B. Interleukin-1 and tumor necrosis factor
C. Prostaglandin and bradykinin
D. Histamine and serotonin
E. Leukotriene and HPETE

The answer is C

14

30

Pathology Exam Test Option - 20121

DIRECTIONS: Each of the numbered items or incomplete statements in this section is followed by answers or by completions of
the statement.
Select the ONE lettered answer or completion that is BEST in each case and fill in the circle c the corresponding letter on the
answer sheet.

1. Which of the following substances is most important in killing bacteria within phagosomes of neutrophils?
(A) Complement
(B) Lactic acid
(C) Opsonin
(D) Oxygen metabolites
(E) Prostaglandins
Answer is D

2. Alcohol keratin is composed of


(A) Desmin
(B) Vimentin
(C) Pre keratin
(D). Neurofilaments

Answer is C

3. Exudation of plasma & leukocytes in acute inflammation is from the ….


(A) Venules
(B) Capillaries
(C) Arterioles
(D) Both b & c

Answer is A

4. A 48-year-old woman has a malignant lymphoma involving lymph nodes in the para-aortic region. She is treated with a chemotherapeutic agent
nolan
which results in the loss of individual neoplastic cells through fragmentation of individual cell nuclei and cytoplasm. Over the next 2 months, the x soju
lymphoma decreases in size, as documented on abdominal CT scans. By which of the following mechanisms has her neoplasm primarily responded to
therapy?
(A) Coagulative necrosis
(B) Mitochondrial poisoning
(C) Phagocytosis
(D) Acute inflammation
(E) Apoptosis
(E) CORRECT

5. A pulmonary abscess is most likely to be associated with which of the following?


(A) Adult respiratory distress syndrome
(B) Asbestosis
(C) Bronchogenic carcinoma
(D) Emphysema
(E) Primary atypical pneumonia Answer is C
6. While in a home improvement center warehouse buying paint, a 35-year-old man hears "Look out below!" and is then struck on
the leg by a falling pallet rack, which strikes him on his left leg in the region of his thigh. The skin is not broken. Within 2 days there is
a 5 x 7 cm purple color to the site of injury. Which of the following substances has most likely accumulated at the site of injury to
produce a yellow-brown color at the site of injury 16 days later?
(A) Lipofuscin
(B) Bilirubin
(C) Melanin
(D) Hemosiderin
(E) Glycogen (D) CORRECT

30

35

(A) antinuclear antibody


(B) elevated C3 concentration
(C) lymphocytosis
(D) monoclonal gammopathy
(E) positive bacterial cultures

Answer is A

34. Crescent formation is characteristic of the following glomerular disease:


(A). Minimal change disease
(B). Rapidly progressive glomerulonephritis
(C) Focal and segmental glomerulosclerosis
(D). Rapidly non progressive glomerulonephritis

Answer is B

35. A 66-year-old woman has the sudden loss of movement on part of the left side of her body. She has smoked a pack of cigarettes
a day for the past 45 years. She has vital signs including T 37.1 C, P 80/minute, RR 16/minute, and BP 160/100 mm Hg. A cerebral
angiogram reveals occlusion of a branch of her middle cerebral artery. Laboratory findings include a hemoglobin A1C of 9%. Which
of the following components of blood lipids is most important in contributing to her disease? (A) Chylomicrons
(B) Lipoprotein lipase
(C) Oxidized LDL
(D) VLDL
(E) HDL cholesterol
(C) CORRECT.

36. Vitamic C deficiency results in


(A) a decreased number of collagen fibers
(B) an increased elastin: collagen ratio
(C) formation of defective collagen fibers
(D) detachment of ribosomes in fibroblasts
(E) insufficient ground substance nolan x soju
Answer is C

37. An autopsy study reveals that evidence for atheroma formation can begin even in children. The gross appearances of the aortas
are recorded and compared with microscopic findings of atheroma formation. Which of the following is most likely to be the first
visible gross evidence for the formation of an atheroma?
(A) Thrombus
(B) Fatty streak
(C) Calcification
(D) Hemorrhage
(E) Exudate
(F) Ulceration
(B) CORRECT.

38. Among the following, transitional cell carcinoma of the urinary bladder most often is associated with
(A) metastases to bone
(B) metastases to the kidneys
(C) metastases to the lungs
(D) metastases to the central nervous system
(E) local extension to surrounding tissuesAnswer is E

39. Disease or infarction of neurological tissue causes it to be replaced by: (A). Fluid
(B). Neuroglia
(C). Proliferation of adjacent nerve cells
(D). Blood vessel

Answer is B

35

36

40. Raynaud disease characteristically


(A) occurs almost entirely in young male cigarette smokers
(B) is associated with atherosclerosis
(C) occurs primarily in tropical climates
(D) is the result of a vasospastic reaction
(E) involves elastic arteries

Answer is D

41. Each of the following pairs is correctly associated EXCEPT


(A) air embolism - deep sea diving
(B) tumor arterial embolism - pulmonary infarct
(C) fat embolism - fractures of long bones
(D) paradoxical embolism - atrial septal defect
(E) venous embolism – postoperative state

Answer is B

42. In von Hippel-Lindau disease, the multiple vascular lesions involving several organs are known as
(A) Necrotizing arteriolitis
(B) Giant cell arteritis
( C) Thromboangitis obliterans
(D) Cavernous hemangiomas
(E) Teleangiectases

The answer is D

43. Flat small vegetations in the cusps of both tricuspid and mitral valves are seen in:
(A). Viral myocarditis
(B). Libmann Sachs endocarditis
(C). Rheumatic carditis
(D). Infective endocarditis nolan x soju
The answer is D

44. A 50-year-old malnourished alcoholic with severe dental caries complains of malaise and right chest
pain of about 2 weeks’ duration. He is febrile and has a cough productive of foul, purulent sputum. A roentgenogram
of the chest shows a fluid-filled cavity, 2 cm in diameter, in the upper lobe of the right lung. The lesion is most likely
(A) an abscess
(B) pulmonary infarct
(C) tuberculosis
(D) bronchiectasis
(E) squamous cell carcinoma

Answer is A

45. A 63-year-old man has had increasing exercise intolerance for the past 5 years so that he now becomes short of breath upon
climbing a single flight of stairs. Laboratory studies have shown fasting blood glucose measurements from 145 to 210 mg/dL for the
past 25 years, but he has not sought medical treatment. If he dies suddenly, which of the following is most likely to be the immediate
cause of death?
(A) Myocardial infarction
(B) Nodular glomerulosclerosis
(C) Cerebral hemorrhage
(D) Hyperosmolar coma
(E) Right lower leg gangrene
(A) CORRECT.

46. Enzyme that protects the brain from free radical injury is:
(A). Myeloperoxidase
(B). Superoxide dismutase
(C). MAO

36

45

Pathology – Exam
Test Option 20123
Directions: Each question below contains five suggested answers. Choose the one best response to each question.

1. A 50-year-old woman with a history of unstable angina suffers an acute myocardial infarction. Thrombolytic
therapy with tissue plasminogen activator (tPA) is administered to restore coronary blood flow early in the course
of this myocardial infarction. In spite of this therapy, the degree of myocardial fiber injury may increase because
of which of the following cellular abnormalities?
A. Cytoskeletal intermediate filament loss
B. A decreased intracellular pH from anaerobic glycolysis
C. An increase in toxic oxygen radicals
D. Mitochondrial swelling
E. Nuclear chromatin clumping and decreased protein synthesisThe answer is C

2. Which of the following pigments will accumulate as the result of peroxidation of cell membranes?
A. bilirubin
B. lipochrome
C. melanin
D. hemomelanin
E. hemosiderinThe answer is B
nolan x soju
3. Severe cell injury due to ischemia is characterized by
A. increased cellular capacity for oxidative phosphorylation
B. a loss of intracellular calcium
C. shrinkage of mitochondria
D. decreased intracellular pH
E. decreased intracellular lactateThe answer is D

4. The earliest histological features of coagulative necrosis of cardiac muscle can be detected by light
microscopyA. 30-40- minutes following vascular occlusion
B. 6 hours following vascular occlusion
C. 12 hours following vascular occlusion
D. 24 hours following vascular occlusion
E. 36 hours following vascular occlusionThe answer is C

5. Following coronary artery occlusion, the earliest morphologic finding that closely corresponds to severe loss of
membrane integrity and irreversible cell injury is
A. swelling and vacuolization of mitochondria and the appearance of large mitochondrial densities
B. cell swelling
C. depletion of cellular glycogen stores (as demonstrated by PAS stain)
D. blebs on cell membranes
E. clumping and condensation of nuclear chromatin

45
7. An Amputated lower limb from a diabetic patient showing black
discoloration of the skin and soft tissues with areas of yellowish exudates is
characterized as: a) Neoplasia
b) Gangrenous Necrosis
c) Coagulopathy
d) Hemosiderosis
e) Gas gangrene

8. Focal fat necrosis, with flecks of chalky tan-white material seen in the
omentum is most often associated with the following: a) Gangrenous
appendicitis
b) Chronic salpingitis
c) Acute pancreatitis
d) Hepatitis
e) Acute gastritis

9. In a 60 year old male, Gangrene of toes is most likely associated with:


a) Diabetes Mellitus
b) Heart Failure
c) Blunt force trauma
d) AIDS
e) Type III hypersensitivity reaction

10. The presence of Columnar Epithelium with Goblet cells in the Lower
Oesophagus is most consistent with:
a) Dysplasia
nolan x soju
b) Hyperplasia
c) Carcinoma
d) Ischaemia
e) Metaplasia

11. Which of the following processes explain the appearance of Calcium


deposition in Tuberculous Lymph Nodes:
a) Dystrophic calcification
b) Apoptosis
c) Hypercalcaemia
d) Metastatic calcification
e) Neoplastic change

12. Lipofuscin deposition is most likely to result from:


a) Nuclear pyknosis
b) Myocardial fiber hypertrophy
c) Coagulative necrosis
d) Autophagocytosis
e) Anaerobic glycolysis

13. Which of the following cellular changes is most likely to present irreversible
cellular injury:
a) Epithelial dysplasia
b) Cytoplasmic fatty metamorphosis
c) Nuclear pyknosis
d) Atrophy
e) Anaerobic Glycolysis

14. Which of the following is an Anti-Oxidant:


a) Glutathione Peroxidase
b) Catalase
c) Hydrogen peroxide
d) NADPH oxidase
e) Myeloperoxidase

15. Which of the following is the most likely pathologic alteration following
occlusion of the left middle cerebral artery by a sterile thrombus: a)
Cerebral softening from liquefactive necrosis
b) Pale infarction with coagulative necrosis
c) Predominantly the loss of glial cells
d) Recovery of damaged neurons if the vascular supply is re-established
e) Wet gangrene with secondary bacterial infection

16. The action of putrefactive bacteria on necrotic tissue results in:


a) Coagulation
b) Infarction
c) Gangrene
d) Embolism nolan x soju
e) Caseation

17. Which of the following type of necrosis is most commonly associated with
ischaemic injury:
a) Coagulation Necrosis
b) Liquefaction Necrosis
c) Caseous Necrosis
d) Fat Necrosis
e) Gangrenous Necrosis

18. Enzymes involved in the inactivation of free radicals include:


a) Superoxide dismutase
b) Vitamin E
c) Both A and B
d) Neither A nor B
e) α-antitrypsin

19. Hepatocytes are an example of:


a) Permenant cells
b) Stable cells
c) Labile cells
d) Metaplasia
e) Mesenchymal cells

20. Calcium may play a role in cell injury by:


a) Causing ATP depletion
b) Activating phospholipases
c) Inducing autophagocytosis
d) Regulating pyknosis
e) Reducing intracellular pH

21. A common manifestation of sublethal cell injury (reversible injury) in organs


such as the heart and liver is:
a) Glycogen accumulation
b) Fatty change
c) Calcium deposition
d) Apoptosis
e) Deposition of melanin

22. An area of keratinizing squamous epithelium lining a major bronchus is an


example of:
a) Heterotopia
b) Metaplasia
c) Dysplasia
d) Atrophy
e) Neoplasia nolan x soju
23. The enzymes responsible for liquefaction in an abscess are derived mainly
from:
a) Tissue
b) Serum
c) Lymph
d) Neutrophils
e) Lymphocytes

24. Each of the following is an example of hyperplasia except:


a) Enlargement of one kidney after surgical removal of the other kidney
b) Changes in the thyroid gland in response to increased demand for thyroid
hormones
c) Changes in breast tissue during pregnancy
d) Enlargement of lymph nodes during a viral infection
e) Enlargement of skeletal muscles after weight training

25. Select the wrong statement. Apoptosis:


a) Occurs in singles cells or in small cluster of cells
b) Is programmed cell death
c) Is seen in toxic or viral hepatitis
d) Appears cheesy in appearance
e) Does not elicit an inflammatory reaction

26. Intracellular system(s) sensitive to cell injury is/are:


a) Aerobic respiration
b) Maintenance of the integrity of cell membrane
c) Synthesis of protein
d) Integrity of genetic apparatus
e) All of the above

27. Which of the following can undergo apoptosis:


a) Cells infected with virus
b) Cells with DNA damage
c) Increased oxidant within the cell
d) None of the above
e) All of the above

28. Apoptosis:
a) Massive necrosis
b) Foreign body phagocytosis
c) Lipoprotein synthesis
d) Programmed cell death
e) Wet gangrene

29. Enzymes involved in the inactivation of free radicals include:


a) Superoxide dismutase nolan x soju
b) Vitamin E
c) Both A an B
d) Neither A nor B
e) α-antitrypsin

30. The following is true about hyperplasia except:


a) Can be physiological
b) Is a precancerous condition
c) Is reversible
d) Is due to excess hormone stimulation
e) Can be associated with hypertrophy

31. Early clumping of nuclear chromatin is most closely associated with:


a) Reduced intracellular pH
b) Increased intracellular pH
c) Denatured proteins and RNA loss
d) Decreased DNA synthesis
e) Release of lysosomal enzymes

40. Digestion of tissue with soap formation and calcification is characteristic of:
a) Coagulation Necrosis
b) Caseous Necrosis
c) Fibrinoid Necrosis
d) Liquefaction Necrosis
e) Enzymatic Fat Necrosis

41. All of the following findings represent changes seen within cells that
represent reversible cellular injury, except: a) Fatty change in
hepatocytes
b) Neuronal cell swelling
c) Skeletal muscle fiber anaerobic glycolysis
d) Renal tubular cell nuclear pyknosis
e) Kupffer cell iron deposition

42. Which is most likely to happen following a stroke with loss of blood supply
to a lobe of the brain:
a) Cerebral softening from liquefactive necrosis
b) Pale infarction with coagulative necrosis
c) Predominantly the loss of glial cells
d) Recover of damaged neurons if the vascular supply is restablished
e) Wet gangrene with secondary bacterial infection

43. The best example of Dystrophic Calcification is seen in a:


a) 55 year old woman with metastases from breast carcinoma &
hypercalcemia
b) Healing granuloma in a 41 year old man with pulmonary tuberculosis
c) Gangrenous lower extremity in a 50 year old woman with diabetes mellitus
d) 62 year old woman with a recent cerebral infarction
e) Abscess of the left fallopian tube in a 19 year old woman with Neisseria
Gonnorrheae Infection

44. Which of the following cellular changes represents the best evidence for
irreversible cellular injury:
a) Epithelial dysplasia
b) Cytoplasmic fatty metamorphosis
c) Nuclear pyknosis
d) Atrophy
e) Anaerobic glycolysis

45. The presence of differentiated columnar epithelium with goblet cells in lower
esophagus is consistent with:
a) Dysplasia
b) Hyperplasia
c) Carcinoma
d) Ischemia
e) Metaplasia
46. Which of the following will show hypertrophy:
a) The uterine myometrium in pregnancy
b) The female breast at puberty
c) The liver following partial resection
d) The ovary following menopause
e) The cervix with chronic inflammation

47. Which of the following is most likely to give rise to Metaplasia:


a) Tanning of the skin following sunlight exposure
b) Lactation following pregnancy
c) Vitamin A deficiency
d) Acute Myocardial Infarction
e) Acute Tubular Necrosis

48. Which of the following is deposited in myocardium in advanced age:


a) Hemosidrin
b) Lipochrome
c) Glycogen
d) Cholesterol
e) Calcium Salts

49. Which of the following tissues is most likely to be least affected by Ischemia:
a) Skeletal muscle
b) Small intestinal epithelium
c) Retina
d) Myocardium nolan x soju
e) Hippocampus

50. An amputated foot of a diabetic will most likely show:


a) Neoplasia
b) Gangrenous Necrosis
c) Coagulopathy
d) Hemosiderosis
e) Caseation

51. Which of the following processes explains the appearance of calcium


deposition in a tuberculous lymph node: a) Dystrophic calcification
b) Apoptosis
c) Hypercalcemia
d) Metastatic calcification
e) Neoplastic change

52. Which of the following types of necrosis is grossly opaque & "chalk" white:
a) Coagulation necrosis
b) Liquefaction
c) Caseous necrosis
d) Fat necrosis
e) Gangrenous necrosis

53. Dystrophic Calcification is most closely associated with:


a) Hypercalcemia
b) Necrosis
c) Chronic irritation
d) Diminished blood supply
e) Increased workload

54. Deposition of Calcium in dead or dying tissue is:


a) Dystrophic calcification
b) Metastatic calcification
c) Both A and B
d) Neither A nor B

55. Which of the following types of necrosis is characterized by amorphous


granular debris:
a) Coagulation Necrosis
b) Liquifaction Necrosis
c) Caseous Necrosis
d) Fat Necrosis
e) Gangrenous Necrosis

56. Hypertrophy is most closely associated with:


a) Hypercalcemia
b) Necrosis nolan x soju
c) Chronic Irritation
d) Diminished Blood Supply
e) Increased Work Load

64. Two days after myocardial infarction, histology of the heart will show:
a) Fibroblasts and Collagen
b) Granulation Tissue
c) Necrotic Muscle & Neutrophils
d) Granulamatous Inflammation
e) Aneurysmal Dilation

65. Cigarette Smoking does not cause:


a) Squamous Metaplasia
b) Smooth Muscle Hyperplasia
c) Defective Ciliary Action
d) Damage of Airway
e) Inhibition of Alveolar Leukocytes & Macrophages
66. Which of the following is responsible for gangrene of lower extremeties:
a) Obesity
b) Hyperlipidemia
c) Cigarette Smoking
d) Diabetes Mellitus
e) Hypercalcemia

67. Which of the following is not a feature of reversible cell injury:


a) Swollen cell
b) Swollen mitochondria
c) Pyknotic Nucleus
d) Dilated Endoplasmic
e) Cell surface blebs

68. Changes of color in Gangrene is due to:


a) Deposition of amyloid
b) Breakdown of hemoglobin
c) Deposition of melanin
d) Deposition of lipofusin
e) Deposition of calcium salts

69. Which of the following is not a feature of reversible cell injury:


a) Increased DNA synthesis
b) Reduction of ATP
c) Failure of Sodium Pump
d) Influx of Calcium nolan x soju
e) Accumulation of Metabolites

70. Which of the following is a feature of Colliquative Necrosis:


a) Complete loss of architecture
b) Cyst formation
c) Occurs in tissues with high water content
d) A and C
e) All of the above

71. Which of the following is (are) true about apoptosis:


a) It is programmed cell death
b) It requires cell energy
c) Induces a severe inflammatory reaction
d) Both A and B
e) Both B and C

72. Calcium may play a role in cell injury by:


a) Causing ATP depletion
b) Activating Phospholipases
c) Inducing Autophagocytosis
d) Regulating Pyknosis

73. Irreversible cellular changes include all of the following except:


a) Hydropic Change
b) Pyknosis
c) Karyorrhexis
d) Karyolysis

74. Abnormal pigmentation is seen in the following except:


a) Conn's Syndrome
b) Pernicious Anemia
c) Tattooing
d) Addison's Disease
e) Pregnancy

75. The following are true about Apoptosis except:


a) It is programmed cell death
b) Affects a group of cells
c) Is energy dependent
d) There is chromatin condensation
e) Does not elicit an inflammatory reaction

76. Which of the following is not a feature of Necrosis: nolan x soju


a) Karyolysis
b) Karyorrhecsis
c) Cytoplasmic Basophilia
d) Pyknosis
e) Loss of RNA

77. Which of the following is not a feature of Coagulative Necrosis:


a) Deposition of Fibrinolysis
b) Tissues are firm and swollen
c) Ghost outline of cell are seen
d) Architecture is maintained
e) The tissue is deeply eosinophilic

78. Which of the following is most likely associated with Caseation Necrosis:
a) Diabetic Gangrene
b) Gas Gangrene
c) Myocardial Infarction
d) Inpissated Pus
e) Splenic Infarction

79. The following are known causes of Atrophy except:


a) Aging
b) Starvation
c) Increased trophic hormone secretion
d) Immobilization
e) Denervation

80. Dystrophic Calcification is:


a) Reversible
b) Due to reduction of pH in tissues
c) Causes renal failure
d) Associated with primary hyperparathyroidism
e) Occurs in dead parasites in the body

81. Which of the following pathologic changes would be considered irreversible?


a) Fatty change in liver cells
b) Hydrophic vacuolization or renal tubular epithelial cells
c) Karyoloysis in myocardial cells
d) Glycogen deposition in hepatocyte nuclei
e) Hyperplasia of the breast during pregnancy

82. Squamous Metaplasia can occur in all of the following sites except: a)
Nose
b) Salivary Gland Ducts
c) Jejunum
d) Renal Pelvis
e) Gall Bladder
nolan x soju
83. Colliquative Necrosis occurs in:
a) Spleen
b) Liver
c) Heart
d) Brain
e) Kidneys

84. The following are features of Colliquative Necrosis:


a) Complete loss of architecture
b) Cyst formation
c) Occurs in tissues with high water content
d) A & C
e) All of the above

85. Which of the following is significant in Reperfusion Injury:


a) Generation of Free Radicals
b) Decreased pH
c) Cytokines released by damaged hypoxic cells
d) Both A & B
e) Both A & C

86. Reperfusion Injury is due to:


a) Generation of Oxygen Free Radicals
b) Additional recruitment of polymorphs by cytokines from hepatic cells
c) Both A & B
d) None of the above

87. Which of the following is a feature of irreversible cell injury:


a) Swelling of Endoplasmic Reticulum
b) Formation of Blebs
c) Karyolysis of Nuclei
d) Fatty Change
e) Mitochondrial Swelling

88. A common manifestation of sublethal cell injury seen in organs such as the
heart and liver is:
a) Glycogen accumulation
b) Fatty change
c) Calcium deposition
d) Apoptosis
e) Deposition of Amyloid

89. In the process of necrosis, a reduction in the size of the nucleus and a
condensation of nuclear material is known as:
a) Karyolysis
b) Pyknophrasia
c) Karyorrhexis
d) Pyknosis nolan x soju
e) Metachromasia

90. A cardinal sign of Inflammation is:


a) Vasoconstriction
b) Redness
c) Leukocyte Margiation
d) Vasodilation
e) Slowing of the Circulation

91. After initiating an acute inflammatory event, the third in a sequence of


changes in vascular flow is: a) Vasoconstriction
b) Redness
c) Leukocyte Margiation
d) Vasodilation
e) Slowing of the Circulation

92. Leukocytes capable of significant Phagocytosis include:


a) Lymphocyte
b) Plasma Cell
c) Both A and B
d) Neither A nor B
93. Which of the following is most associated with Acute Inflammation:
a) Neutrophils
b) Connective Tissue
c) Macrophages
d) Granulation Tissue
e) Granuloma

94. Inflammatory conditions which have a large number of eosinophils in the


exudates include:
a) Asthma
b) Parasitic disease
c) Both A and B
d) Neither A nor B

95. By definition, which cells are involved in organization:


a) Kidney tubular and Liver Parenchymal cells
b) Macrophages and Endothelial Cells
c) Endothelial Cells and Fibroblasts
d) Fibroblasts and Fat Cells

96. Phagocytosis is enhanced by:


a) C5a
b) Bradykinin
c) Lysozyme
d) C3b nolan x soju
e) Serotonin

97. The tumor of inflammation is due primarily to:


a) Arteriolar Dilation
b) Venous Dilation
c) Capillary Dilation
d) Increased Intracellular Fluid
e) Increased Extracellular Fluid

98. A mediator of acute inflammation that causes increased vascular


permeability and pain is: a) Endotoxin
b) Complement
c) Histamine
d) Bradykinin
e) Endogenous Pyrogen

99. Multinucleated Giant Cells of the foreign body type origin from:
a) Nuclear Division of Granulocytes
b) Atypical Regeneration of Epithelium
c) Megakaryocytes
d) Fusion of Macrophages
e) Multiplication of Nuclei Surrounding Fibrocytes
100. Histamine is thought to be the direct cause of: a) Leukocytosis
b) Emigration
c) Phagocytosis
d) Increased Vascular Permeability
e) All of the Above
.
101 An Abscess is best defined as:
a) A local defect in the surface of a tissue
b) An abnormal accumulation between two surfaces
c) Any area of tissue necrosis
d) A localized collection of pus
e) An epithelium-lined sac filled with viscous fluid

102. Mediators of fever in inflammation include:


a) Macrophage product similar to interleukin I
b) Prostaglandins
c) Both A and B
d) Neither A nor B

103. Phagocytes from an individual deficient in myeloperoxiase (MPO)


will most likely be less effective in: a) Killing
Bacteria
b) Recognizing Bacteria
c) Marginating in vessels
d) Undergoing Chemotaxis
e) Engulfing Bacteria

104. A purulent exudates is generally characterized by the presence of:


a) Mucous
b) Macrophages and connective tissue
c) Neutrophils and necrotic debris
d) Serous Fluid
e) Precipitated Protein
nolan x soju
105. Granuloma formation is most frequently associated with: a)
Acute inflammation
b) The Healing Process
c) Wound Contraction
d) Fibroblasts and Neovasularization
e) A persistent irritant

106. Which of the following best describes a granuloma: a) A


small nodule of granulation tissue
b) A tumor composed of granulocytes
c) A small hard mass of fibroblasts and collagen
d) Inflammation primarily composed of lymphocytes
e) An aggregate of activated macrophages

107. The most characteristic feature of granulation tissue is the:


a) Resemblance to a Granuloma
b) Growth of Fibroblasts and New Capillaries
c) Character of the Exudate
d) A form of Gangrenous Necrosis
e) Presence of Monocytes and Fibroblasts
.

108 Which of the following is the most efficient killer of bacteria in neutrophils:
a) Spontaneous dismutation of O2 to H2O
b) Conversion of H2O2 to HOCl by Myeloperoxiase
c) Oxidative metabolism giving rise to OH and O2
d) Hyrolyzing Bacterial Coated with Lysozyme
e) Action of Hydrolytic Enzymes

109. The emigration and accumulation of neutrophils in the acute


inflammation reaction is primarily the result of: a) Active hyperemia
b) Hydrostatic Pressure
c) Increased Microvascular Permeability
d) Chemotaxis
e) None of the above

110. Which of the following is expressed on endothelial cells in acute


inflammation for leukocyte adhesion: a) Interferon Gamma
b) Hageman Factor
c) Lysozyme
d) E-selectin
e) Prostacyclin

111. Fever and Leukocytes in which most leukocytes are neutrophils typical
of: a) Acute Bacterial Infection
b) Acute Viral Infection nolan x soju
c) Chronic Viral Infection
d) Parasitemia
e) Foreign Body Giant Cell Reaction

112. Epitheloid Cells and Langerhans Giant Cells in granulomas are


derived from:
a) Neutrophils
b) Eosinophils
c) Mast Cells
d) Macrophage
e) Lymphocytes

113. Prostaglandins are formed from Arachidonic Acid through the action
of which enzyme pathway? a) Cyclooxygenase
b) Lipoxygenase
c) Myeloperoxidase
d) Phospholipase A
e) Glutathione Reductase

114. The cavity of an Abscess contains: a) Caseous Necrosis


b) Hyalin
c) Giant Cells
.
d) Pus
e) Granulation Tissue

115 Transudate can be found in the following settings except:


a) Congestive Heart Failure
b) Nephrotic Syndrome
c) Superior Venacaval Obstruction
d) Pericarditis
e) None of the Above

116. The diagnostic feature of granuloma is the presence of:


a) Giant cells
b) Epitheloid cells
c) Lymphocytes
d) Fibrosis
e) Caeseation

117. The following have a granuloma except:


a) T.B
b) Syphilis
c) Leprosy
d) Schistosomiasis
e) Staphylococcal Infection

118. Formation of the epitheloid cells is mediated by: a) IL-1


b) IL-2
c) Histamine nolan x soju
d) Gamma Interferon
e) Serotonin

119. The first inflammatory cell to show at the site of acute inflammation is the:
a) Monocyte
b) B-lymphocyte
c) T-lymphocyte
d) Neutrophils
e) Mast cell

120. The following are true about an exudate except: a) It can be present
as edema
b) It has a high protein content
c) It has a specific gravity ≥ 1.020
d) It may contain inflammatory cells
e) It can be due to congestive heart failure

121. Which of the following is the most powerful chemotactic agent: a)


Histamine
b) Prostaglandin
c) Hageman Factor
d) Bradykinin
.
e) Complement C5a
122. A biopsy of a region of acute inflammation would most likely reveal all of
the following except:
a) Fibrinous exudate
b) Neutrophils
c) Edema
d) Necrosis
e) Fibrous connective tissue

123. Which of the following chemical mediators is most important in the


development of Granulomatous Inflammation:
a) Interferon gamma
b) Bradykinin
c) Complement C5a
d) Histamine
e) Prostaglandin E2

124. Following an attack of chest infection for three days, a patient becomes very
ill and his chest x-rays shows a 3cm rounded density. He is most likely
having: a) Hypertrophic scar
b) Abscess formation
c) Regeneration
d) Bronchogenic carcinoma
e) Progression to chronic inflammation

125. Which of the following cells is the most important in the development of
tuberculous granuloma:
nolan x soju
a) Macrophages
b) Fibroblast
c) Neutrophil
d) Mast Cell
e) Platelet
126. Which of the following inflammatory processes would cause interstitial
fibrosis and nodules in the lungs of a person who inhaled silica dust:
a) Neutrophilic infiltration with release of leukotrienes
b) Foreign body giant cell formation
c) Plasma cell production of immunoglobin
d) Histamine release by mast cells
e) Release of growth factors by macrophages

127. Which of the following mediators are predominantly responsible for pain in
acute inflammation:
a) Complement C3b and IgG
b) Interleukin-1 and tumour necrosis factor
c) Histamine and Serotonin
d) Prostaglandin and Bradykinin
e) Leukotriene and E-selectin
128. Which of the following cell types is most likely to be most characteristic of a
foreign body reaction:
a) Mast cell
b) Eosinophil
c) Giant cell
d) Neutrophils
e) Plasma cell

129. Which of the following is needed for anti-microbial killing during


Phagocytosis:
a) Glutathione peroxidase
b) C3b
c) Interleukin 1
d) NADPH oxidase
e) Myeloperoxidase

130. Deficiency of which of the following will reduce phagocytosis by


neutrophils:
a) C3a
b) Leukotriene A4
c) C3b
d) Leukotriene B4
nolan x soju
e) C5a
131. All of the following are true of granulation tissue except: a) Contains
epitheloid cells
b) Contains myofibroblast
c) Contains proliferating capillaries
d) Important in wound contraction
e) Contains fibronectin

132. Matrix components in early granulation tissue include:


a) Fibronectin
b) Type III collagen
c) Both A and B
d) Neither A nor B
e) Granuloma

133. Apoptosis does not induce inflammation because: a) The injury is too
mild to induce inflammation
b) Phagocytic cells secrete cytokines that inhibit inflammation
c) The process occurs only in avascular tissues
d) Apoptosis occurs only in immune deficient tissues
e) The patient is using antibiotics
134. Osponization is the:
a) Formation of free radicals
b) Degradation of bacteria by lysozymes
c) Engulfment of antigen by leukocytes
d) Coating of antigen by antibodies
e) Processing of antigens by antigen presenting cells

135. Which of the following is not a characteristic of a Granuloma: a)


Macrophages
b) Giant Cells
c) Polymorphonuclear Leukocytes
d) Lymphocytes
e) Epitheloid Cells

136. Which of the following has a large number of eosinophils in exudates:


a) Sarcoidosis
b) Bronchiectasis
c) Syphilis
d) Bronchial Asthma
e) Chronic Bronchitis

137. Which of the following are involved in organization: a) Endothelial


cells and osteocytes
b) Endothelial cells and chondroblasts
nolan x soju
c) Endothelial cells and fibroblasts
d) Fibroblasts and fat cells
e) Fibroblasts and plasma cells

138. The tumour of inflammation is due primarily to:


a) Arteriolar dilation
b) Venous dilation
c) Capillary dilation
d) Increased intracellular fluid
e) Increased extracellular fluid

139. One of the following is a Chemotactic Factor:


a) Histamine
b) Bradykinin
c) Interleukin 1
d) Leukotriene B4
e) Serotonin

140. Phagocytosis is enhanced by:


a) C1
b) Bradykinin
c) Lysozyme
d) Osponin
e) Serotonin

141. Which one of the following is a harmful effect of acute inflammation:


a) Dilution of toxins
b) Phagocytosis
c) Formation of fibrin
d) Swelling of tissue
e) Stimulation of immune response

142. The high protein content of the inflammatory exudates is mainly due to:
a) Protein released from dead tissue
b) Production of protein by inflammatory cells
c) Increased capillary permeability
d) Increased capillary hydrostatic pressure
e) Increased blood flow in the site of inflammation

143. Which of the following is expected to be found in sputum of an asthmatic


child:
a) IgE antibodies
b) Activate macrophages nolan x soju
c) Eosinophils
d) Foreign body giant cell
e) Asbestos bodies

144. Phagocytosis is enhanced by: a) Prostaglandin E2


b) Bradykinin
c) Lysozymes
d) Osponin
e) Serotonin

145. Which of the following are involved in organization: a) Liver


parenchymal cells
b) Macrophages and endothelial cells
c) Endothelial cells and fibroblasts
d) Fibroblasts and fat cells
e) Neurons and glial cells

146. Which part of the microcirculation is most consistently involved in the


permeability changes and exudation of acute inflammation:
a) Small arteries
b) Arterioles
c) Capillaries
d) Venules
e) Veins

147. Chronic Inflammation is characterized by the presence of: a)


Polymorphs, monocytes and exudate
b) Plasma cells, lymphocytes, fibroblasts and monocytes
c) Plasma cells, polymorphs, lymphocytes and monocytes
d) Plasma cells, basophils, eosinophils, fibroblasts and monocytes

148. The following are true of macrophages except: a) They are phagocytic
cells
b) They produce interleukins
c) Are antigen-presenting cells
d) They are derived from blood monocytes
e) They are the main cells in chronic inflammation
149. Which of the following is true about an Exudate: a) Occurs in heart
failure
b) Is due to low plasma oncotic pressure
c) Has a low specific gravity
d) Has a high protein content
e) Is due to lymphatic obstruction

150. The most important factor in the formation of acute inflammatory exudates
is:
a) Increased Hydrostatic Pressure
b) Increased Blood Flow
nolan x soju
c) Chemotaxis
d) Increased Vascular Permeability
e) Lymphatic Obstruction

151. Increased Vascular Permeability is due to: a) Damage to endothelial


cells
b) Widening of gaps between endothelial cells
c) Slowing of blood flow
d) Both A & B
e) Both A & C

152. Infertility and short stature in a woman with a 45x karyotype are typical of:
a) Rlinefelter Syndrome
b) Turner Syndrome
c) Down Syndrome
d) Achondroplasia Dwarfism
e) Cystic Fibrosis

153. Down Syndrome is:


a) Trisomy 21
b) Trisomy 18
c) Trisomy 13
d) Autosomal Dominant
e) Mothers are less than 30 years old

154. Mutations in mitochondrial genes are: a) Inherited from the father


b) Common cause of disease
c) Inherited from the mother
d) Autosomal Recessive
e) Affects only males

155. There are no Y-linked disorders because:


a) There are no mutations occurring in the Y chromosome
b) The affected males die before adulthood
c) Affected males are infertile
d) Defects in the Y chromosome are autosomal dominant
e) None of the above
163. The primary role of which of the following in the closure of wounds healing
by second intention: a) Neutrophils
b) Connective Tissue
c) Macrophages
d) Granulation Tissue
e) Granuloma

164. The process of regeneration:


a) Does not restore prior function
b) Invariability leads to scar formation
c) Refers to healing by proliferation of stromal elements nolan x soju
d) Occurs in tissues composed of labile and stable cells
e) Is synonymous with repair

165. The following factors delay healing of damaged tissue:


a) Scurvy
b) Ionizing Radiation
c) Immobilization
d) Infections
e) Glucocorticoids

166. Repair of which of the following involves regeneration and connective tissue
repair:
a) Bone fracture
b) Renal tubular necrosis
c) Both A and B
d) Neither A nor B

167. Which of the following does not promote fracture healing:


a) Steroid therapy
b) Adequate blood supply
c) Immobilization
d) Adequate Vitamin C supply
168. Neurogenic, Septic and Cardiogenic shock are all characterized by:
a) Equal prognosis if untreated
b) Need for blood transfusion
c) Peripheral vasodilation at onset
d) Normal blood volume at early stage
e) Progression to irreversible shock

169. The causes of Hypovolemic shock do not include: a) Severe vomiting


b) Severe diarrhea
c) Extensive burning
d) Myocardial Infarction
e) Severe Trauma

170. Following a hypotensive shock which lasted for hours, which of the
following tissues is most likely to withstand the ischemia: a) Skeletal
muscle
b) Small Intestinal Epithelium
c) Retina
d) Myocardium
e) Hippocampus

171. What type of shock is most associated with severe burns: a)


Cardiogenic shock
b) Hypovolemic shock
c) Septic shock
nolan x soju
d) Neurogenic shock

172. The causes of Hypovolemic shock do not include: a) Severe vomiting


b) Severe diarrhea
c) Blood Loss
d) Acute hemorrhagic pancreatitis
e) Extensive Burning

173. The following are true about arterial thrombi except: a) Are usually
due to endothelial injury
b) Can be precipitated by atherosclerosis
c) Grow in the direction of blood flow
d) Lead to ischemia and infarction
e) Commonly cause systemic emboli

174. The following are true regarding venous thrombi except: a)


Commonly are due to stasis
b) Always cause infarction
c) Grow in the direction of blood flow
d) Can cause pulmonary emboli
e) Commonly occur in the deep leg veins
175. Thrombosis can be caused by all of the following except: a)
Endothelial injury
b) Turbulent blood flow
c) Leiden mutation
d) Factor VIII deficiency
e) Atrial Fibrillation

176. Which of the following are sequelae to thrombosis: a) Embolization


b) Organization and Recanalization
c) Propagation
d) Both A and B
e) All of the above

177. Which one of the following does not predispose to thrombosis: a)


Endothelial damage
b) Vascular Stasis
c) Thrombocytopenia
d) Formation of aggregates
e) Activation of the coagulation mechanism

178. Which one of the following is not a cause of thrombosis: a) Damage nolan x soju
of endothelial lining
b) Aneurysms
c) Polycythenia
d) Dissiminated Carcinomatosis
e) High level of antithrombin III

179. The following are associated with DIC except:


a) Widespread deposition of fibrin with microcirculation
b) Consumption of coagulation factors
c) Haemorrhagic diathesis (Bleeding Tendency)
d) Thrombocytosis (Increased Platelet Count)
e) Activation of Plasminogen

180. Disseminated Intravascular Coagulation is associated with the following


except:
a) Microangiopathic hemolytic anemia
b) Thrombocytosis
c) Low fibrinogen level
d) Disseminated malignancy
e) Crush injuries

181. The following predispose to deep vein thrombosis except:


a) Malnutrition
b) Cancer
c) Factor VIII deficiency
d) Abdominal Surgery
e) Immobilization

182. The following are true of Thrombi except: a) Are formed from blood
consitituents
b) Always cause complete vascular occlusion and infarction
c) Are complicated by embolization
d) May undergo dystrophic calcification

183. Which of the following is not associated with Thrombosis:


a) Cancer
b) Prolonged bed rest
c) Sickle cell anemia
d) Hemophilia
e) Tissue damage

184. Fat emboli can be precipitated by:


a) Excessive fats intake
b) Rupture of an atheromatous plaque
c) Rupture of the uterus
d) Fracture of the femur
e) Changes in atmospheric pressure

185. Paradoxial emboli are:


a) Commonly pulmonary emboli
b) Are caused by myocardial infarction
c) Due to the presence of an interventricular on inter atrial defect
d) Due to sudden changes in the atmospheric pressure
e) Caused by trauma

186. Pulmonary emboli have each of the following characteristics except:


a) Commonly arise in leg veins
b) Pass through the right heart before reaching the lung
c) Consistently cause infarction in the lung
d) Are usually thrombotic emboli

187. A detached intravascular solid, liquid or gas carried to a site distant from its
point of origin is most associated with:
a) Ascites
b) Petechiae
c) Infarction
d) Emboli
e) Hematoma

188. Infarction of the spleen is usually due to:


a) Hypersplenism
b) Congestion
c) Arterial Embolism
d) Deposition of connective tissue and pigment
e) Venous Thrombosis

189. In an area of myocardial infarction, which of the following will be most


effective in promoting healing: a) Histamine
b) Immunoglobulin G
c) Complement C3b
d) Leukotriene B4
e) VEGF (vascular endothelial growth factor)

190. Red Infarcts develop in:


a) Spleen
b) Liver
c) Kidney
d) Adrenal Glands
e) Intestine

nolan x soju
191. Which of the following is most significant in the diagnosis of early acute
myocardial infarction:
a) ESR
b) ASO titre
c) Troponin
d) LDH
e) Total CK

192. Which of the following is not a major risk factor for atherosclerosis:
a) Family history
b) Hyperlipidemia
c) Physical inactivity
d) Diabetes
e) Hypertension

193. Organs which are less susceptible than others to infarction because of a dual
blood supply include:
a) Liver and Lungs
b) Liver and Kidney
c) Lung and Spleen
d) Spleen and Kidney
e) Pancreas and Lung
194. Which cells are targeted by HIV:
a) CD4 + cells
b) Plasma cells
c) Macrophages
d) Both A & B
e) Both A & C

195. Asthma is an example of:


a) Type I hypersensitivity reaction
b) Type II hypersensitivity reaction
c) Type III hypersensitivity reaction
d) Type IV hypersensitivity reaction
e) Type V hypersensitivity reaction

196. IgE plays a major role in:


a) Type I hypersensitivity reaction
b) Type II hypersensitivity reaction
c) Type III hypersensitivity reaction
d) Type IV hypersensitivity reaction
e) Type V hypersensitivity reaction

197. Acute graft rejection is caused by:


a) Type I hypersensitivity reaction
b) Type II hypersensitivity reaction
c) Type III hypersensitivity reaction nolan x soju
d) Type IV hypersensitivity reaction
e) Type V hypersensitivity reaction

205. Which of the following cell types releases chemical mediators


responsible for allergic rhinitis:
a) Neutrophils
b) Mast Cells
c) CD4+ Cells
d) NK Cells
e) Macrophages

206. Which of the following chemical mediators is most likely to be


involved in anaphylaxis following intravenous penicillin treatment:
a) Interleukin I
b) Bradykinin
c) Complement C5a
d) Histamine
e) Thromboxane
207. Which of the following cellular process is most likely to occur in the
myocardium of a patient with myocardial infarction who is being
treated with fibrinolytic agents:
a) Apoptosis
b) Free Radical Injury
c) Fatty Changes
d) Accumulation of Lipofuscin
e) Accumulation of Amyloid

208. Which of the following will predict a better prognosis for breast
cancer: a) The tumor is small in size
b) No metastasis are found in sampled lymph node
c) Numerous mitosis are seen
d) Tumor shows mild hyperchromatism
e) Tumor cells show marked pleomorphism

209. Which of the following is characteristic of neoplasm: a)


Recurrence following excision
b) Rapid increase in size
c) Sensitivity to radiation therapy
d) Uncontrolled (autonomous growth)
e) Necrosis

210. Widespread Oedema is likely in: a) Lymphatic Obstruction


b) Capillary Damage
c) Inflammations
nolan x soju
d) Hypoalbuminemia
e) Cancer of the Stomach

211. Acute Pulmonary Oedema of the heart failure results from: a)


Increased Plasma Colloid Osmotic Pressure
b) Decreased Plasma Colloid Osmotic Pressure
c) Increased Vascular Permeability
d) Increased Vascular Hydrostatic Pressure
e) Increased Blood Viscosity
212. DIC following extensive surgery is due to:
a) Entrance of tissue thromboplastin into circulation
b) Hypoxia, acidosis and shock coexisting with extensive surgery
c) Protein C Deficiency
d) Both A and B
e) Both B and C

213. Which of the following is not a feature of stem cells: a) Capable


of self renewal
b) Capable of linkage generation
c) They are the predominant population in a tissue
d) They belong to the terminal differentiation compartment
e) Both A and B
214. Which of the following is not a feature of benign tumors: a)
Grow Slowly
b) Well Demonstrated
c) Infiltrate Adjacent Cells
d) Can Secrete Hormones
e) Named by adding suffix 'oma' to type of cell

215. Which of the following malignant tumors has the best prognosis:
a) Hepatocellular Carcinoma
b) Renal Cell Carcinoma
c) Basal Cell Carcinoma
d) Squamous Cell Carcinoma of Cervix
e) Squamous Cell Carcinoma of Skin

216. The contents of a blister is an example of which type of exudate:


a) Serous
b) Fibrinous
c) Purulent
d) Cellular
e) Haemaorrhagic

217. Reduced Plasma Oncotic Pressure is the most important cause of


generalized edema in:
a) Congestive Heart Failure
b) Lung Cancer
nolan x soju
c) Constrictive Pericarditis
d) Head Trauma
e) Nephrotic Syndrome

218. Which of the following diseases is characterized by noncaseating


granulomas?
a) Sarcoidosis
b) Tuberculosis
c) Histoplasmosis
d) Diptheria
e) Measles

234. Which of the following causes Lymphatic Oedema?


a) Sarcoidosis
b) Schistosomiasis
c) Filariasis
d) Syphilis
e) Leishmaniasis

235. Which of the following does not cause Hypokalaemia?


a) Diueretics
b) Vomiting
c) Diarrhoea
d) Addison's Disease
e) Metabolic Alkalosis

236. The following are associated with Cachexia except: a) Malignancy


b) Anemia
c) Wasting
d) Hypoplasia
e) Tumour Necrosis Factor

237. Acute effects of Ionizing Radiation do not include: a) Leukemia


b) Anemia
c) Skin Desquamation
d) Pulmonary Oedema
e) Vomitting

238. Primary stage of Syphilis is best diagnosed by: a) VDRL Test


b) RPR Test
c) Widal Test
d) Smear seen under dark ground illumination
e) TPHA

239. Granuloma Formation is not a feature of: a) Tuberculoid Leprosy


b) Sarcoidosis
c) Schistosoma manson infection nolan x soju
d) Tuberculosis
e) Cerebral Malaria

240. Pathological effects of Shock do not include: a) Gastric Ulceration


b) D.I.C.
c) Acute Renal Tubular Necrosis
d) Metabolic Alkalosis
e) Adult Respiratory Distress Syndrome

241. Peg-Shaped (Hutchinson's Teeth) are characteristically seen in babies


born to mothers suffering from:
a) Mucosal Leishmaniasis
b) Syphilis
c) Sarcoidosis
d) Intestinal Tuberculosis
e) AIDS
242. In Amoebiasis:
a) Condylomata Lata is a feature
b) Granuloma Formulation is characteristic
c) The small intestine is always involved
d) The perianal skin may be affected
e) There is an intense infiltration in the lesions by polymorphs

243. Consequences of Necrosis do not include: a) Dystrophic


Calcification
b) Resolution
c) Inflammatory Reaction
d) Fibrosis
e) Increased Cellular Enzymes in the Blood

244. In Lepromatous Leprosy:


a) Epitheloid Granuloma Formation is characteristic
b) There is loss of the Rete Ridges of the epidermis
c) Bacilli are scanty
d) Spontaneous cure may occur
e) Lepromin test is negative

245. One of the following is not true about bilharzial portal hypertension:
a) Splenomegaly is of the congestive type
b) Iron deficiency anemia is a complication
c) Periportal fibrosis is the cause
d) There is an increased incidence of hepatocellular carcinoma nolan x soju
e) Hepatic failure is not an early complication

246. Gummas most often appear in:


a) Spleen
b) Pancreas
c) Liver and Testes
d) Kidneys
e) Thyroid

247. Hepar Lobatum is liver showing:


a) Miliary Tuberculosis
b) Multiple Gummas and Fibrosis
c) Periportal Fibrosis
d) Amoebic Abscess
e) Sarcoid Granulomas

You might also like